Download as pdf or txt
Download as pdf or txt
You are on page 1of 72

INSTA CURRENT AFFAIRS

QUIZ

JANUARY 2024

WWW.INSIGHTSONINDIA.COM INSIGHTSIAS
INSTA CURRENT AFFAIRS QUIZ
Table of Contents

1. ECONOMY .........................................................................................................................................2

2. ECOLOGY AND ENVIRONMENT ........................................................................................................ 14

3. GEOGRAPHY .................................................................................................................................... 25

4. GOVERNMENT SCHEMES AND PROGRAMMES ................................................................................. 34

5. SCIENCE AND TECHNOLOGY............................................................................................................. 39

6. INTERNATIONAL RELATIONS AND ORGANISATIONS .......................................................................... 46

7. POLITY ............................................................................................................................................. 53

8. HISTORY, ART AND CULTURE............................................................................................................ 60

9. AGRICULTURE .................................................................................................................................. 62

10. DEFENCE AND SECURITY............................................................................................................... 65

11. REPORTS AND INDICES ................................................................................................................. 66

12. MAPS / PLACES............................................................................................................................. 67

www.insightsonindia.com 1
INSTA CURRENT AFFAIRS QUIZ

1. Economy
1) Consider the following statements.
1. The Coinage Act, 2011 gives the central government the power to design and mint coins in various
denominations.
2. All of India’s currency note printing presses are owned by the Government of India.
3. The first banknote issued in independent India was the Re 1 note that replaced the portrait of King
George with the symbol of the Lion Capital of the Ashoka Pillar at Sarnath.
How many of the above statements is/are correct?
a) Only one
b) Only two
c) All three
d) None

Solution: b)

Statement 2 is incorrect.

The Coinage Act, 2011 gives the central government the power to design and mint coins in various
denominations. In the case of coins, the role of the RBI is limited to the distribution of coins that are supplied by
the central government.
The government decides on the quantity of coins to be minted on the basis of indents received from the RBI on a
yearly basis, the central bank says. Coins are minted in four mints owned by the Government of India
in Mumbai, Hyderabad, Kolkata and Noida.

Two of India’s currency note printing presses (in Nasik and Dewas) are owned by the Government of India; two
others (in Mysore and Salboni) are owned by the RBI through its wholly owned subsidiary, Bharatiya Reserve Bank
Note Mudran Ltd (BRBNML).

Ashoka Pillar Banknotes: The first banknote issued in independent India was the Re 1 note issued in 1949.
While retaining the existing design, the new banknotes replaced the portrait of King George with the symbol of
the Lion Capital of the Ashoka Pillar at Sarnath in the watermark window.

2) Consider the following statements regarding Reserve currency.


1. A reserve currency is widely used in cross-border transactions.
2. It is commonly held as reserves by central banks.
3. Till now U.S. dollar is the only currency recognised as international reserve currency.
How many of the above statements is/are correct?
a) Only one
b) Only two
c) All three
d) None

Solution: b)

Statement 3 is incorrect.

A reserve currency refers to any currency that is widely used in cross-border transactions and is commonly
held as reserves by central banks.

Other currencies such as the British pound and the French franc have served as international reserve
currencies in the past. It should be noted that it is the currencies of economic superpowers that have usually
ended up being used as the global reserve currency. As the economic clout of these countries waned, their
currencies faced a similar downfall. This was the case, for example, with the British pound which was gradually
www.insightsonindia.com 2
INSTA CURRENT AFFAIRS QUIZ
replaced by the U.S. dollar as Britain lost its status as a global economic superpower in the first half of the 20th
century.

3) Consider the following statements regarding Foreign Contribution (Regulation) Act.


1. The Act prohibits receipt of foreign funds by candidates for elections, journalists and media broadcast
companies and organisations of a political nature.
2. The law was enacted during the Economic Reforms in 1991.
3. Once granted, FCRA registration is valid for one year and NGOs have to apply for renewal before the
expiry of registration.
How many of the above statements is/are correct?
a) Only one
b) Only two
c) All three
d) None

Solution: a)

Only Statement 1 is correct.

FCRA seeks to “regulate the acceptance and utilisation of foreign contribution or foreign hospitality by certain
individuals or associations or companies and to prohibit [such] acceptance and utilisation…for any activities
detrimental to the national interest…”
The law was enacted during the Emergency in 1976.

The Act prohibits receipt of foreign funds by candidates for elections, journalists or newspaper and media
broadcast companies, judges and government servants, members of legislature and political parties or their
office-bearers, and organisations of a political nature.

FCRA registrations are granted to individuals or associations that have definite cultural, economic, educational,
religious, and social programmes.
Following the application, the MHA makes inquiries through the Intelligence Bureau into the antecedents of the
applicant, and accordingly processes the application. The MHA is required to approve or reject the application
within 90 days — failing which it is expected to inform the NGO of the reasons for the same.

Once granted, FCRA registration is valid for five years. NGOs are expected to apply for renewal within six
months of the date of expiry of registration. In case of failure to apply for renewal, the registration is deemed to
have expired.

The government reserves the right to cancel the FCRA registration of any NGO if it finds it to be in violation of
the Act. Registration can be cancelled for a range of reasons including, if “in the opinion of the Central
Government, it is necessary in the public interest to cancel the certificate”. Once the registration of an NGO is
cancelled, it is not eligible for re-registration for three years. All orders of the government can be challenged in
the High Court.

4) Consider the following statements.


1. The U.S. central bank usually increases the supply of dollars to tackle economic downturns and also to
fund the U.S. government’s expenditures.
2. The global acceptability of the U.S. dollar has primarily been attributed to the popularity of U.S. assets
among investors.
3. U.S. has been running a persistent trade surplus for decades now.
How many of the above statements is/are correct?
a) Only one
b) Only two
c) All three
d) None
www.insightsonindia.com 3
INSTA CURRENT AFFAIRS QUIZ

Solution: b)

Statement 3 is incorrect.

The U.S. central bank usually increases the supply of dollars through various means to tackle economic
downturns and also to fund the U.S. government’s expenditures. But it should be noted that the U.S. Federal
Reserve is not the only central bank in the world that has been debasing its currency by engaging in
expansionary monetary policy over several decades. Other countries have also been expanding their respective
money supplies to address their domestic economic problems. As long as the U.S. does not debase its currency
at a faster pace than other countries, the dollar may manage to hold its value against other currencies and hence
its reserve currency status may not come under serious threat.

The global acceptability of the U.S. dollar has primarily been attributed to the popularity of U.S. assets among
investors. It should be noted that the U.S. has been running a persistent trade deficit for decades now (in fact
the last time the U.S. ran a trade surplus was way back in 1975). That is, the value of its imports has for a long
time exceeded the value of its exports to the rest of the world. The excess dollars that the rest of the world
accumulates due to the U.S.’s trade deficit has been invested in U.S. assets such as in debt securities issued by
the US government.

5) Consider the following statements regarding US debt ceiling.


1. Debt ceiling is the maximum amount of debt the federal government is allowed to accumulate in order
to fulfil its financial obligations.
2. The debt ceiling was introduced in order to make it easier for the executive to operate without having
to turn to the Congress every time it wants to spend.
3. The US Government would not enter into a default even if the debt ceiling is breached.
How many of the above statements is/are correct?
a) Only one
b) Only two
c) All three
d) None

Solution: b)

Statement 3 is incorrect.

The “debt ceiling” or “debt limit” is the maximum amount of debt the federal government is allowed to
accumulate in order to fulfil its financial obligations.

The government typically spends more than it earns. This means that it needs to borrow in order to pay for things
like social security, Medicare, military and other salaries, interest on the national debt, tax refunds, etc.
The debt limit was introduced in 1917 – when the US entered World War I – and according to the US Treasury
Department, the Congress has increased or suspended the debt limit 78 times since 1960. As of 2023, the debt
ceiling stands at $ 31.4 trillion.

Constitutionally, the Congress controls the government’s purse strings. The debt ceiling was introduced in order
to make it easier for the executive to operate without having to turn to the Congress every time it wants to
spend – it allowed the government to borrow as required as long as it kept under the debt limit, which has to be
approved by the Congress.

If the debt ceiling is breached, the Treasury Department would be unable to make payments when they are due –
meaning that the US would enter into a default.

6) Consider the following statements regarding RoDTEP scheme.

www.insightsonindia.com 4
INSTA CURRENT AFFAIRS QUIZ
1. RoDTEP scheme is aimed at ensuring that no domestic taxes are added on to goods’ shipments meant
for export.
2. The scheme replaced the Merchandise Export from India Scheme (MEIS).
3. Any item which is restricted is not entitled for the RoDTEP benefits.
How many of the above statements is/are correct?
a) Only one
b) Only two
c) All three
d) None

Solution: c)

The government is trying to work out a mechanism to bestow tax remission benefits on exports of sugar, which
had been placed in the ‘restricted’ category in mid-2022, and is considering a demand from tea exporters to
raise their tax remission rates.

The Remission of Duties and Taxes on Export Products (RoDTEP) scheme, launched in January 2021, is aimed
at ensuring that no domestic taxes are added on to goods’ shipments meant for export. Authorities had denied
benefits under the scheme to sugar exporters after the sweetener was removed from the ‘free’ category in June
2022 as a pre-emptive measure to boost domestic availability and avoid a spurt in prices.

“Any item which is restricted is not entitled for the RoDTEP benefits. So once sugar was brought into the
restricted list, it has been prevented from getting RoDTEP,”

The scheme was announced as a replacement for the Merchandise Export from India Scheme (MEIS), which was
not compliant with the rules of the World Trade Organisation.

Source

7) The computation of poverty in terms of monthly per capita consumption expenditure based on mixed
reference period (MRP) was recommended by
a) Dandekar Committee
b) Lakdawala Committee
c) Tendulkar Committee
d) Alagh committee

Solution: c)

The Tendulkar Committee for the first time recommended use of implicit prices derived from quantity and value
data collected in household consumer expenditure surveys for computing and updating the poverty lines.
Tendulkar Committee developed a methodology using implicit prices for estimating state wise poverty lines for
the year 2004-05. Using these poverty lines and distribution of monthly per capita consumption expenditure
based on mixed reference period (MRP), the Tendulkar Committee estimated poverty ratios for the year 2004-05

8) Consider the following statements.


1. Logistics includes only the management and coordination of the flow of goods and not services.
2. Logistics Performance Index 2023 is released by IMF.
3. In India, Unorganized sector amounts to less than 10% of the logistics sector.
How many of the above statements is/are correct?
a) Only one
b) Only two
c) All three
d) None

Solution: d)
www.insightsonindia.com 5
INSTA CURRENT AFFAIRS QUIZ

Logistics refers to the management and coordination of the flow of goods, services, and information from the
point of origin to the point of consumption. It involves various activities such as transportation, storage,
handling, and distribution to ensure the efficient movement of products within the supply chain.

India has ranked 38 out of 139 countries in the World Bank’s Logistics Performance Index 2023 (2018 rank of India
was 44)

The unorganized sector amounts to over 90% of the logistics sector.

Source

9) Consider the following statements regarding Cash Reserve Ratio (CRR)


1. Cash Reserve Ratio (CRR) is the percentage of deposits which a bank must keep as cash reserves with
the banks themselves.
2. This is a legal requirement and is binding on the banks.
3. Banks don’t earn returns on money parked as CRR.
4. The Central Bank controls the liquidity in the Banking system through CRR.
How many of the above statements is/are correct?
a) Only one
b) Only two
c) Only three
d) All four

Solution: c)

Statement 1 is incorrect

Cash Reserve Ratio (CRR)


• The RBI decides a certain percentage of deposits which every bank must keep as reserves. This is done to
ensure that no bank is ‘over lending’. This is a legal requirement and is binding on the banks. This is called
the ‘Required Reserve Ratio’ or the ‘Reserve Ratio’ or ‘Cash Reserve Ratio’ (CRR).
• Cash Reserve Ratio (CRR) = Percentage of deposits which a bank must keep as cash reserves.
• In CRR, the cash reserve is maintained by the banks with the Reserve Bank of India.
• In the case of SLR, the securities are kept with the banks themselves, which they need to maintain in the form
of liquid assets.
• Banks don’t earn returns on money parked as CRR.
• The Central Bank controls the liquidity in the Banking system through CRR.

10) Consider the following statements regarding laws of demand and supply.
1. The law of demand asserts that as prices rise, the quantity demanded for an economic good decreases.
2. Producers are willing to supply more at higher prices because the increased selling price compensates
for the elevated opportunity cost associated with each additional unit sold.
Which of the above statements is/are incorrect?
a) 1 only
b) 2 only
c) Both 1 and 2
d) Neither 1 nor 2

Solution: d)

The law of demand states that, if all other factors remain equal, the higher the price of a good, the less people will
demand that good. In other words, the higher the price, the lower the quantity demanded.
The amount of a good that buyers purchase at a higher price is less because as the price of a good goes up, so
does the opportunity cost of buying that good.
www.insightsonindia.com 6
INSTA CURRENT AFFAIRS QUIZ
The law of supply says that at higher prices, sellers will supply more of an economic good. From the seller’s
perspective, the opportunity cost of each additional unit that they sell tends to be higher and higher. Producers
supply more at a higher price because the higher selling price justifies the higher opportunity cost of each
additional unit sold.

11) Consider the following statements regarding evergreening of loans.


1. It refers to the practice where a lender extends new credit to borrowers who are struggling to meet
their debt obligations.
2. RBI has mandated that banks and financial institutions must go for evergreening of loans to ensure a
healthy financial system.
Which of the above statements is/are correct?
a) 1 only
b) 2 only
c) Both 1 and 2
d) Neither 1 nor 2

Solution: a)

The Reserve Bank of India (RBI) has strengthened regulations for Regulated Entities (RE), including banks and
financial institutions, to curb the evergreening of loans through investments in Alternative Investment Funds
(AIFs).

What is the evergreening of loans?


• It refers to the practice where a lender extends new credit to borrowers who are struggling to meet
their debt obligations. Instead of addressing the underlying financial issues, the lender provides additional
funds, essentially giving the appearance that the borrower’s financial situation is stable.
• This can create a temporary illusion of creditworthiness but may lead to a cycle of increasing debt and
delayed resolution of financial difficulties. Regulatory authorities often aim to prevent evergreening to
ensure a transparent and healthy financial system.

12) Which of the following types of funds are considered as Alternative Investment Funds?
1. Venture Capital Funds
2. Angel Funds
3. Debt Funds
4. Hedge Funds
5. Social Venture Funds
How many of the above options is/are correct?
a) Only two
b) Only three
c) Only four
d) All five

Solution: d)

www.insightsonindia.com 7
INSTA CURRENT AFFAIRS QUIZ

13) Consider the following statements.


1. Venture Capital Fund: Invest into early-stage, emerging growth companies.
2. Hedge Fund: Invest in anything that give profitable returns in all types of market conditions.
3. Real Estate Fund: Make equity or debt investments into stable cash flow generative properties.
How many of the above statements is/are correct?
a) Only one
b) Only two
c) All three
d) None

Solution: c)

www.insightsonindia.com 8
INSTA CURRENT AFFAIRS QUIZ

14) Consider the following statements.


1. A sovereign credit rating is an independent assessment of a country’s creditworthiness, that evaluates
the country’s ability to repay debt without default.
2. A good credit rating influences borrowing costs in global financial markets.
3. In India RBI regulates credit rating agencies.
How many of the above statements is/are correct?
a) Only one
b) Only two
c) All three
d) None

Solution: b)

Statement 3 is incorrect.

What is Credit Credit rating is an assessment of the creditworthiness of a borrower, including


Rating? individuals, companies, or countries.
A sovereign credit rating is an independent assessment of a country’s
creditworthiness. It evaluates the country’s ability to repay debt without
What is default, impacting the risk associated with investing in its debt instruments.
Sovereign Credit
Rating? Key agencies include Standard & Poor’s, Moody’s, and Fitch
Ratings, considering economic indicators, fiscal policies, political stability, and
trade position.
Obtaining a good credit rating is crucial for accessing funding in the
Importance of international bond market and attracting foreign investments. It also
Credit Rating influences borrowing costs in global financial markets, allowing countries with
higher ratings to borrow at lower interest rates.
SEBI in India regulates credit rating agencies, including CRISIL, CARE, ICRA,
Credit Rating in
Acuité Ratings, Brickwork Rating, India Rating and Research Pvt. Ltd, and
India
Infomerics Valuation and Rating Pvt. Ltd.

Source

15) Consider the following statements regarding Tokenization.


www.insightsonindia.com 9
INSTA CURRENT AFFAIRS QUIZ
1. Tokenization is the process of replacing sensitive data with a non-sensitive equivalent.
2. Card-on-file tokenisation (CoFT) is a security measure for users opting for digital payments, replacing
the practice of merchants storing card details with specially created tokens.
3. Tokens are random strings of characters that have no meaningful value if breached.
How many of the above statements is/are correct?
a) Only one
b) Only two
c) All three
d) None

Solution: c)

The Reserve Bank of India (RBI) has expanded the scope of card-on-file tokenisation (CoFT) to include card issuing
banks and institutions directly, moving beyond services provided through merchants.
CoFT is a security measure for users opting for digital payments, replacing the practice of merchants storing
card details with specially created tokens.

Tokenization is the process of replacing sensitive data with a non-sensitive equivalent, called a token. Tokens
are random strings of characters that have no meaningful value if breached. They serve as a reference to the
original data, but cannot be used to guess those values.

Source

16) Consider the following statements regarding loan write-off.


1. Writing off a loan essentially means it will no longer be counted as an asset, by the bank.
2. The amount so written off does not reduce the bank’s tax liability.
3. After the write-off, banks are not supposed to continue their efforts to recover the loan.
How many of the above statements are correct?
a) Only one
b) Only two
c) All three
d) None

Solution: a)

Only statement 1 is correct.

Writing off a loan essentially means it will no longer be counted as an asset. By writing off loans, a bank can
reduce the level of non-performing assets (NPAs) on its books. An additional benefit is that the amount so
written off reduces the bank’s tax liability.

The bank writes off a loan after the borrower has defaulted on the loan repayment and there is a very low chance
of recovery. The lender then moves the defaulted loan, or NPA, out of the assets side and reports the amount as a
loss.

After the write-off, banks are supposed to continue their efforts to recover the loan using various options. They
have to make provisioning as well. The tax liability will also come down as the written-off amount is reduced from
the profit.
However, the chances of recovery from written-off loans are very low.

17) Consider the following statements regarding fund raising by Indian companies.
1. Indian companies' fund raising via dollar bonds hit an all-time high in 2023.
2. Elevated global yields usually discourage fund raising via dollar bonds and instead attract borrowers to
foreign currency loans.
3. Foreign currency loans are benchmarked to floating interest rates.
www.insightsonindia.com 10
INSTA CURRENT AFFAIRS QUIZ
How many of the above statements is/are correct?
a) Only one
b) Only two
c) All three
d) None

Solution: b)

Statement 1 is incorrect.

Indian companies' fundraising via dollar bonds hit a 14-year low in 2023, as elevated global yields discouraged
borrowers who moved to securing foreign currency loans instead.

Throughout 2023 global interest rate environment was not favourable and the cost of raising dollars hedged into
rupees was comparatively higher than (accessing) domestic liquidity, so Indian issuers chose to borrow locally.

Multiple rate hikes by the U.S. Federal Reserve and anticipation of prolonged higher rates drove an increase in
bond yields worldwide.
Instead, Indian companies leaned on foreign currency loans, which are benchmarked to floating interest rates.

Source

18) Consider the following statements regarding Multilateral Development Banks (MDBs).
1. Multilateral development banks (MDBs) originated in the aftermath of World War II to rebuild war-
ravaged nations.
2. They do not impose any conditions while providing loans.
3. International Monetary Fund and World Bank are the only two Multilateral development banks in
operation throughout the world.
How many of the above statements is/are correct?
a) Only one
b) Only two
c) All three
d) None

Solution: a)

Only Statement 1 is correct.

• Multilateral development banks (MDBs) originated in the aftermath of World War II to rebuild war-ravaged
nations and stabilize the global financial system.
• Today, MDBs fund infrastructure, energy, education, and environmental sustainability in developing
countries.
• While commercial banks seek to make profits on loans and other financial services, the goal of MDBs is to
issue grants and low-cost loans to improve the economic conditions of impoverished or developing nations.
• MDBs now operate throughout the world and control trillions of dollars in assets.
• E.g., International Monetary Fund, World Bank, Asian Development Bank etc.

Issues: Currently, MDBs are dominated by developed Countries (WB by USA, IMF by EU and ADB by Japan); they
impose conditionality in loans; there is a lack of Transparency and Accountability.

19) In India, which of the following entities can provide microfinance?


1. Small finance banks
2. Non-banking financial companies (NBFCs)
3. Not-for profits organisations
4. Public Sector Banks
www.insightsonindia.com 11
INSTA CURRENT AFFAIRS QUIZ
How many of the above options are correct?
a) Only one
b) Only two
c) Only three
d) All four

Solution: d)

The number of loan accounts for the microfinance industry increased to 136.3 million in FY23, from 123.9 million
in FY22, posting a year-on-year (y-o-y) growth of 10%. Lender-wise figures showed that NBFCs registered highest
y-o-y growth of 23%, followed by NBFC-MFIs at 15%, not-for profits or NFPs at 6%, banks at 6%, and small
finance banks at 5%.

20) Securities and Exchange Board of India (SEBI) acts as a regulatory authority for which of the following?
1. Primary Market
2. Secondary Market
3. Mutual Funds.
4. Foreign Institutional Investment
5. Credit Rating agencies in India
How many of the above options is/are correct?
a) Only two
b) Only three
c) Only four
d) All five

Solution: d)

21) Consider the following statements regarding Securities and Exchange Board of India (SEBI).
1. SEBI is a quasi-legislative body that has the power to conduct inquiries, pass ruling and impose
penalties.
2. It cannot regulate the money pooling scheme worth more than Rs. 100 crores.
3. It educates investors about securities markets and their intermediaries.
How many of the above statements is/are correct?
a) Only one
b) Only two
c) All three
d) None

Solution: b)

Statement 2 is incorrect.

www.insightsonindia.com 12
INSTA CURRENT AFFAIRS QUIZ

www.insightsonindia.com 13
INSTA CURRENT AFFAIRS QUIZ

2. Ecology and Environment


1) Consider the following statements regarding Natural gas.
1. Natural gas is used to power appliances like stoves and furnaces.
2. Natural gas is a relatively clean burning fossil fuel, resulting in fewer emissions of nearly all types of air
pollutants and CO2 when compared with burning coal or petroleum products.
3. Extraction of natural gas from the ground does not involve release of any greenhouse gas.
How many of the above statements is/are correct?
a) Only one
b) Only two
c) All three
d) None

Solution: b)

Statement 3 is incorrect.

Natural gas, extracted from the earth, powers appliances like stoves and furnaces.

Natural gas is mainly methane, which is a greenhouse gas – capable of trapping significant amounts of heat in the
atmosphere. Natural gas is a relatively clean burning fossil fuel, resulting in fewer emissions of nearly all types
of air pollutants and CO2 when compared with burning coal or petroleum products to produce an equal amount
of energy.

Extraction of natural gas from the ground also involves release of certain greenhouse gases.

2) Consider the following statements regarding Permafrost.


1. Permafrost is essentially any ground that stays frozen for at least two straight months.
2. Permafrost is composed of a combination of soil, rocks and sand that are held together by ice.
3. Permafrost regions are always covered with snow.
How many of the above statements is/are correct?
a) Only one
b) Only two
c) All three
d) None

Solution: a)

Only Statement 2 is correct.

Permafrost is essentially any ground that stays frozen — 0 degree Celsius or lower — for at least two years
straight. These permanently frozen grounds are often found in Arctic regions such as Greenland, Alaska (the
United States), Canada, Russia and Eastern Europe.

Permafrost is composed of “a combination of soil, rocks and sand that are held together by ice. The soil and ice
in permafrost stay frozen all year long.” However, although the ground remains perennially frozen, permafrost
regions aren’t always covered with snow.

As the Arctic is getting warmer nearly four times as fast as the rest of the planet due to climate change,
permafrost is thawing rapidly. Thawing permafrost can severely impact the planet. One of its most dangerous
consequences is the release of greenhouse gases into the atmosphere.

3) Consider the following statements regarding Loss and Damage (L&D) fund, that was agreed at COP28.
1. Loss and Damage (L&D) fund will be hosted by the World Bank.
www.insightsonindia.com 14
INSTA CURRENT AFFAIRS QUIZ
2. It is aligned with UNFCCC and the Paris Agreement.
3. The contributions by the developed countries are mandatory.
How many of the above statements is/are correct?
a) Only one
b) Only two
c) All three
d) None

Solution: b)

Statement 3 is incorrect.

COP28 countries agreed to launch the Loss and Damage (L&D) fund, hosted by
Loss and the World Bank for four years, aligning with UNFCCC and the Paris Agreement. All
Damage developing countries are eligible, and contributions are voluntary, with a specific
(L&D) Fund percentage earmarked for Least Developed Countries and Small Island Developing
States.

Source

4) Consider the following statements regarding Powering Past Coal Alliance (PPCA).
1. PPCA is a coalition involving governments, businesses, and organizations, focusing on transitioning
from unabated coal power to clean energy.
2. India is part of PPCA as it has committed to phasing out of coal production.
Which of the above statements is/are correct?
a) 1 only
b) 2 only
c) Both 1 and 2
d) Neither 1 nor 2

Solution: a)

PPCA, a coalition involving governments, businesses, and organizations, focuses


Powering
on transitioning from unabated coal power to clean energy. At COP28, PPCA
Past Coal
welcomed new national and subnational governments, advocating for cleaner
Alliance
energy alternatives. India is not part of PPCA as it has not committed to phasing
(PPCA)
out of coal.

5) Consider the following statements regarding Buildings Breakthrough Initiative.


1. The goal of the Buildings Breakthrough Initiative is to make near-zero emissions and resilient buildings
the new normal by 2030.
2. The initiative was launched by India.
3. It is hosted by the Global Alliance for Buildings and Construction.
How many of the above statements is/are correct?
a) Only one
b) Only two
c) All three
d) None

Solution: b)

Statement 2 is incorrect.

www.insightsonindia.com 15
INSTA CURRENT AFFAIRS QUIZ
The Governments of France and Morocco, together with the UN Environment Programme (UNEP), launched the
Buildings Breakthrough today at COP28, which will see countries joining forces to accelerate the transformation
of the sector – which accounts for 21 per cent of global greenhouse gas emissions – with a view to making near-
zero emissions and climate resilient buildings the new normal by 2030.

The Buildings Breakthrough is part of the Breakthrough Agenda, which provides a framework for countries,
businesses and civil society to join up and strengthen their actions every year in key emitting sectors, through a
coalition of leading public, private and public-private global initiatives.

It is coordinated under the auspices of the UNEP-hosted Global Alliance for Buildings and
Construction (GlobalABC).

Source

6) Consider the following statements regarding Global River Cities Alliance (GRCA).
1. Global River Cities Alliance is led by the National Mission for Clean Ganga (NMCG) under the Ministry of
Jal Shakti.
2. It focuses on river conservation and sustainable water management.
3. Asian Development Bank and Asian Infrastructure and Investment Bank acts as international funding
agencies.
How many of the above statements is/are correct?
a) Only one
b) Only two
c) All three
d) None

Solution: c)

www.insightsonindia.com 16
INSTA CURRENT AFFAIRS QUIZ

7) Consider the following statements regarding Pangolakha Wildlife Sanctuary, that was recently in news.
1. The Pangalokha Wildlife Sanctuary is situated at the tri-junction of Sikkim, Bengal, and Bhutan.
2. It is the smallest wildlife sanctuary in Sikkim.
3. A tiger was spotted at the Pangolakha Wildlife Sanctuary, which marks the highest elevation in
India where tigers have been observed.
How many of the above statements is/are correct?
a) Only one
b) Only two
c) All three
d) None

Solution: b)

Statement 2 is incorrect.

www.insightsonindia.com 17
INSTA CURRENT AFFAIRS QUIZ
A tiger has been spotted at a record-high elevation of 3,640 meters in the Pangolakha Wildlife Sanctuary in
Sikkim. This marks the highest elevation in India where tigers have been observed, surpassing the previous
record of 3,630 meters in Arunachal Pradesh.

The Pangalokha Wildlife Sanctuary is situated at the tri-junction of Sikkim, Bengal, and Bhutan. The Pangolakha
range in the east separates Sikkim from Bhutan

It is the largest wildlife sanctuary in Sikkim.

It is home to various species, including red pandas, snow leopards, Himalayan musk deer, goral, and black bears.

Source

8) Sharks are ectotherms. What does it mean?


a) If heat loss exceeds heat generation, metabolism increases to make up the loss
b) They maintain a constant body temperature independent of the environment
c) Their body temperature is largely determined by the surrounding water temperature
d) None of the above

Solution: c)

Ectotherm, any so-called cold-blooded animal—that is, any animal whose regulation of
body temperature depends on external sources, such as sunlight or a heated rock surface. The ectotherms include
the fishes, amphibians, reptiles, and invertebrates. The body temperature of an aquatic ectotherm is usually
very close to the temperature of the surrounding water.

9) Consider the following statements regarding Ecotone


1. Estuary is an example of Ecotone.
2. Ecotone is a zone of tension.
3. A well-developed ecotone contains some organisms which are entirely different from that of the
adjoining communities.
How many of the above statements are correct?
a) Only one
b) Only two
c) All three
d) None

Solution: c)

Ecotone is a zone of junction between two or more diverse ecosystems. For e.g. the mangrove forests represent
an ecotone between marine and terrestrial ecosystem. Other examples are - grassland, estuary and river bank.

Characteristics of Ecotone
• It may be very narrow or quite wide.
• It has the conditions intermediate to the adjacent ecosystems. Hence it is a zone of tension.
• It is linear as it shows progressive increase in species composition of one in coming community and a
simultaneous decrease in species of the other outgoing adjoining community.
• A well-developed ecotone contains some organisms which are entirely different from that of the adjoining
communities.

10) Consider the following statements:


1. Natural gas is an odorless, gaseous mixture of hydrocarbons—predominantly made up of methane.
2. It is a non-fossil fuel and renewable source of energy.
Which of the above statements is/are correct?
a) 1 only
www.insightsonindia.com 18
INSTA CURRENT AFFAIRS QUIZ
b) 2 only
c) Both 1 and 2
d) Neither 1 nor 2

Solution: a)

Natural gas is an odorless, gaseous mixture of hydrocarbons—predominantly made up of methane (CH4).


Natural gas is a fossil fuel energy source. Natural gas contains many different compounds. The largest component
of natural gas is methane, a compound with one carbon atom and four hydrogen atoms (CH4). Natural gas also
contains smaller amounts of natural gas liquids (NGLs, which are also hydrocarbon gas liquids), and
nonhydrocarbon gases, such as carbon dioxide and water vapor. We use natural gas as a fuel and to make
materials and chemicals.

11) Consider the following statements.


1. Phytoremediation is a remediation method that uses living organisms like plants, microalgae, and
seaweeds to reduce the concentrations or toxic effects of contaminants in the environment.
2. Metals like cadmium, cobalt, nickel, lead and organic pollutants can be easily removed from the soil
through Phytoremediation.
3. Hyperaccumulator plants slows down the process of Phytoremediation.
How many of the above statements is/are correct?
a) Only one
b) Only two
c) All three
d) None

Solution: a)

Only Statement 1 is correct.

“Phytoremediation”, a remediation method that uses living organisms like plants, microalgae, and seaweeds. One
particular way to remove toxic heavy metals from the soil includes the use of “hyperaccumulator” plants that
absorb these substances from the soil.
Phytoremediation refers to the usage of “hyperaccumulator” plants to absorb the toxic materials present in the
soil and accumulate in their living tissue. Even though most plants do sometimes accumulate toxic substances,
hyperaccumulators have the unusual ability to absorb hundreds or thousands of times greater amounts of
these substances than is normal for most plants.
This process can be used to remove metals like silver, cadmium, cobalt, chromium, copper, mercury, manganese,
molybdenum, nickel, lead and zinc; metalloids such as arsenic and selenium; some radionuclides; and non-
metallic components such as boron. But it cannot be used to remove organic pollutants from the ground due to
metabolic breakdown.

12) Consider the following statements regarding Camelids.


1. A Camelid refers to any of the even-toed ungulates of the family Camelidae.
2. The United Nations has declared 2025 as the International Year of Camelids.
3. Camelids contribute to food security, nutrition, and economic growth, especially for Indigenous
Peoples and local communities.
How many of the above statements is/are correct?
a) Only one
b) Only two
c) All three
d) None

Solution: b)

Statement 2 is incorrect.
www.insightsonindia.com 19
INSTA CURRENT AFFAIRS QUIZ

A Camelid refers to any of the even-toed ungulates of the family Camelidae.

• The United Nations has declared 2024 as the International Year of Camelids to emphasize the significant
importance of camelids, such as alpacas, Bactrian camels, dromedaries, guanacos, llamas, and vicuñas, in the
lives of people worldwide.
• Camelids contribute to food security, nutrition, and economic growth in over 90 countries, especially
for Indigenous Peoples and local communities.
• They play a crucial role in meeting Sustainable Development Goals (SDGs) by providing sources of milk, meat,
fibre, and transportation.

Source

13) Consider the following statements.


1. Valmiki Tiger Reserve (VTR) is located in Andhra Pradesh.
2. Recently Valmiki Tiger Reserve (VTR) was in news due to rapid decline of Tiger population from 2014 to
current year.
Which of the above statements is/are correct?
a) 1 only
b) 2 only
c) Both 1 and 2
d) Neither 1 nor 2

Solution: d)

The rise in tiger numbers at Bihar’s Valmiki Tiger Reserve (VTR) is attributed to a reduction in human
activities, particularly restrictions on illegal mining in the eco-sensitive zone.
The ban on sand and stone mining inside VTR, along with strict limits in its eco-sensitive zone, led to increased
grassland cover.
Tiger numbers in VTR increased from 28 in 2014 to 54 in the current year, with 31 recorded in 2018.
Bihar is also pursuing the establishment of a second tiger reserve in Kaimur district.

Source

14) Consider the following statements regarding Namdapha flying squirrel.


1. The Namdapha flying squirrel that was missing for nearly 40 years was rediscovered in Arunachal
Pradesh.
2. It is classified as Critically Endangered (CR) by the IUCN.
3. It is listed under Schedule I of the Wildlife Protection Act.
How many of the above statements is/are correct?
a) Only one
b) Only two
c) All three
d) None

Solution: b)

Statement 3 is incorrect.

The Namdapha flying squirrel, missing for 42 years, has been rediscovered in Arunachal Pradesh.
Last described in 1981, the species was thought to be extinct or confused with a similar species.

About Namdapha Flying Squirrel

www.insightsonindia.com 20
INSTA CURRENT AFFAIRS QUIZ
It is classified as Critically Endangered (CR) by the IUCN and is listed under Schedule II of the Wildlife Protection
Act. Located in Namdapha National Park, Arunachal Pradesh, it was one of the 25 “most wanted lost”
species targeted by the Global Wildlife Conservation’s “Search for Lost Species” initiative.

Source

15) Which is the only Ape species found in India?


a) Orangutan
b) Gray langur
c) Bonobo
d) Hoolock gibbon

Solution: d)

16) Consider the following statements.


1. The Climate Vulnerable Forum (CVF) is a global partnership of countries most at risk from climate
impacts.
2. Alliance of Small Island States (AOSIS) represents countries that are disproportionately vulnerable to
climate effects, particularly sea level rise and coastal erosion.
3. Umbrella Group includes Africa’s U.N. members that push for additional climate financing.
How many of the above statements are correct?
a) Only one
b) Only two
c) All three
d) None

Solution: b)
www.insightsonindia.com 21
INSTA CURRENT AFFAIRS QUIZ

Statement 3 is incorrect.

UMBRELLA GROUP: This alliance of non-EU developed countries includes Australia, Japan, Russia, and the United
States.

AFRICA GROUP: Africa’s U.N. members will push for additional climate financing, while arguing that expanding
economies need fossil fuels to increase electricity capacity.

CLIMATE VULNERABLE FORUM: Representing countries most at risk from climate impacts, including Bangladesh
and the Maldives.

ALLIANCE OF SMALL ISLAND STATES: The alliance, known by its acronym AOSIS, represents countries that are
disproportionately vulnerable to climate effects, particularly sea level rise and coastal erosion.

17) Consider the following statements regarding Self-pollination.


1. The mechanism ensures the plant’s ability to reproduce without relying on external pollinators like
insects or wind.
2. Here the offspring are genetically identical to the parent plant.
3. Self-pollination ensures reproductive success and also increases genetic variability in plant populations.
How many of the above statements is/are correct?
a) Only one
b) Only two
c) All three
d) None

Solution: b)

Statement 3 is incorrect.

Changes in the environment especially due to anthropogenic activities have limited the population
of pollinators (such as Bees). In such a situation, plants evolve to self-pollinate itself.

What is Self-pollination?
It is a reproductive process in plants where pollen from a flower’s male reproductive organs (anther) fertilizes the
female reproductive organs (stigma) of the same flower or another flower on the same plant.
The mechanism ensures the plant’s ability to reproduce without relying on external pollinators like insects or
wind.

Mechanism of the Self-Pollination:


• Anther and Stigma Proximity: In self-pollinating plants, the anther (producing pollen) is close to the stigma
(receiving pollen) within the same flower.
• Pollen Transfer: Pollen grains are transferred from the anther to the stigma, either by physical contact or
through the plant’s internal mechanisms.
• Fertilization: The pollen tube grows down the style to the ovary, where fertilization takes place, resulting in
the formation of seeds.
• Genetic Similarity: Since both the pollen and the ovule come from the same plant, the offspring
are genetically identical to the parent, leading to limited genetic diversity.

Self-pollination ensures reproductive success, especially in environments with limited pollinators, but it can
also reduce genetic variability in plant populations and risk to the plant-pollinator interaction system.

www.insightsonindia.com 22
INSTA CURRENT AFFAIRS QUIZ

Source

18) Consider the following statements regarding Great Nicobar Island.


1. Indira Point on the southern tip of Great Nicobar Island is India’s southernmost point.
2. The Great Nicobar Island has tropical wet evergreen forests, mountain ranges and coastal plains.
3. The leatherback sea turtle is the island’s flagship species.
How many of the above statements are correct?
a) Only one
b) Only two
c) All three
d) None

Solution: c)

Indira Point on the southern tip of Great Nicobar Island is India’s southernmost point, less than 150 km from the
northernmost island of the Indonesian archipelago.

The Great Nicobar Island has tropical wet evergreen forests, mountain ranges reaching almost 650 m above sea
level, and coastal plains. The leatherback sea turtle is the island’s flagship species.

19) Consider the following statements regarding Laughing gull.


1. Laughing gull is a migratory bird from North America.
2. Recently it was sighted for the first time in India in North-Eastern part of the country.
3. It is known as Laughing gull due its unique laughter-like calls.
How many of the above statements is/are correct?
a) Only one
b) Only two
c) All three
d) None

Solution: b)

Statement 2 is incorrect.

• Laughing gull, a migratory bird from North America, has been sighted for the first time in the country at the
Chittari estuary in Kasaragod district.
• The bird, known for its unique laughter-like calls, has travelled thousands of kilometres from North America
to the coastal region of the State.
• The bird has a dark upper part, black legs, long drooping bill, and a dark smudge on the back of the head.

www.insightsonindia.com 23
INSTA CURRENT AFFAIRS QUIZ

Source

20) Consider the following statements regarding Forest Rights Act (FRA).
1. The Forest Rights Act (FRA) enables tribal communities and forest-dwelling citizens to claim legal rights
over the forest land they have occupied for generations.
2. It gives gram sabhas, or village councils, the right to decide on how best their forest land can be used.
3. Any transfer of forest land for non-forestry use does not require the consent of gram sabhas.
How many of the above statements is/are correct?
a) Only one
b) Only two
c) All three
d) None

Solution: b)

Statement 3 is incorrect.

The Forest Rights Act (FRA), officially known as the Scheduled Tribes and Other Traditional Forest Dwellers
(Recognition of Forest Rights) Act, 2006, is a piece of legislation that enables tribal communities and forest-
dwelling citizens to claim legal rights over the forest land they have occupied for generations but in many cases,
denied a say or authority in its management or free access to forest produce.
It gives gram sabhas, or village councils, the right to decide on how best their forest land can be used. Any
transfer of forest land for non-forestry use now requires their consent.

Source

21) Consider the following statements regarding Nandankanan Zoological Park.


1. It holds the distinction of being the first zoo in India to join the World Association of Zoos &
Aquariums (WAZA).
2. It is recognized for breeding Indian pangolins and white tigers.
3. It is located in Odisha.
How many of the above statements is/are correct?
a) Only one
b) Only two
c) All three
d) None

Solution: c)

About Nandankanan Zoological Park


It is located 15 km from Bhubaneswar, Odisha, and was inaugurated in 1960. It holds the distinction of being
the first zoo in India to join the World Association of Zoos & Aquariums (WAZA), a global alliance dedicated to
animal care and conservation. Recognized for breeding Indian pangolins and white tigers, it is also a breeding
ground for leopards, mouse deer, lions, ratel, and vultures. Nandankanan was the world’s first captive crocodile
breeding centre, successfully breeding gharials in captivity in 1980.

Source

www.insightsonindia.com 24
INSTA CURRENT AFFAIRS QUIZ

3. Geography
1) Consider the following statements.
1. Warmer air can hold more moisture than cooler air.
2. A psychrometer is a device for measuring air humidity.
3. A wet-bulb temperature is the lowest temperature a surface reach when water evaporates from it.
How many of the above statements is/are correct?
a) Only one
b) Only two
c) All three
d) None

Solution: c)

Warmer air can hold more moisture than cooler air.

A more direct way to understand the implications of the relative humidity for your wellbeing is in the form of the
wet-bulb temperature (a.k.a. the adiabatic saturation temperature). It is the lowest temperature a surface – like
your skin – can reach when water evaporates from it. The wet-bulb temperature is equal to the dry-bulb
temperature when the relative humidity is 100%.
A wet-bulb temperature in an environment of 32-35º C or higher can be quickly lethal, even if you are not doing
any physical activity or are in the shade. (At least one study has shown that even a wet-bulb temperature of more
than 29º C can be dangerous.)
A psychrometer is a device for measuring air humidity.

2) Recently the Ministry of Jal Shakti released a document, which is the first such census of water bodies in
India. Consider the following statements in this context.
1. Tamil Nadu has the highest number of lakes in India.
2. Among the total number of utilised water bodies in India, majority is used for irrigation.
3. More than 90% of water bodies can be found in rural regions.
How many of the above statements is/are correct?
a) Only one
b) Only two
c) All three
d) None

Solution: b)

Statement 2 is incorrect.

West Bengal boasts of the highest number of ponds and reservoirs; Andhra Pradesh the highest number of
tanks; and Tamil Nadu the highest number of lakes.

The majority of water bodies serve as resources for fish farming, with their subsequent uses including irrigation,
replenishing groundwater, and providing water for household and drinking needs. Among the total 20,30,040
utilised water bodies, 55.5% (11,26,830) are dedicated to fish farming, 16.5% (3,35,768) to irrigation, 12.1%
(2,44,918) to groundwater replenishment, and 10.1% (2,05,197) to domestic and drinking water needs. The
remaining are employed for recreational, industrial, religious and other purposes.

A substantial 97.1% of water bodies can be found in rural regions; only 2.9% are situated in urban areas. Of
these, 83.7% are in use, while the rest are non-functional or unused due to factors such as construction,
siltation, irreparable damage, and industrial effluents, among others.

3) Consider the following statements.


www.insightsonindia.com 25
INSTA CURRENT AFFAIRS QUIZ
1. The combination of high temperature and low humidity is referred to as the wet bulb temperature.
2. High moisture content in the atmosphere makes it difficult for the sweat to evaporate and bodies to
cool down.
3. Oral rehydration solutions (ORS) can act as lifesavers during heatstroke.
How many of the above statements is/are correct?
a) Only one
b) Only two
c) All three
d) None

Solution: b)

Statement 1 is incorrect.

The combination of high temperature and high humidity, referred to as the wet bulb temperature, is what
makes heatwaves deadly. High moisture content in the atmosphere makes it difficult for the sweat to evaporate
and bodies to cool down, as a result of which the internal body temperature increases sharply, and is often fatal.

Access to drinking water or oral rehydration solutions (ORS). These can be lifesavers in such situations.

4) Consider the following statements.


1. El Niño is characterised by an unusual warming of waters in the eastern equatorial Pacific, which has a
high correlation with warmer summers and weaker monsoon rains in India.
2. During El Niño years, most parts of north, northeast, northwest, and central India experience dry
periods during the June to September monsoon season.
3. Normally, El Nino is good for the northeast monsoon.
How many of the above statements is/are correct?
a) Only one
b) Only two
c) All three
d) None

Solution: c)

El Niño is characterised by an unusual warming of waters in the eastern equatorial Pacific, which has a high
correlation with warmer summers and weaker monsoon rains in India. La Nina is the meteorological opposite.

A map WMO issued on expected rainfall patterns globally during El Niño years showed most parts of north,
northeast, northwest, and central India experiencing dry periods during the June to September monsoon
season. It shows largely wet months during the October to December northeast monsoon season in the
peninsular region.

Normally, El Nino is good for the northeast monsoon but bad for the southwest monsoon but there is no one-to-
one relationship.

5) The annual mean temperature of the world is known to have increased by 1.1 degree Celsius from the
average of the 1850-1900 period. Consider the following statements in this context.
1. Temperature rise over land is much higher than over oceans.
2. The increase in temperatures is more prominent near the equator than in the higher altitudes.
3. Ice traps the least amount of heat and reflects most of the solar radiation when compared with land or
water.
How many of the above statements is/are correct?
a) Only one
b) Only two
c) All three
www.insightsonindia.com 26
INSTA CURRENT AFFAIRS QUIZ
d) None

Solution: b)

Statement 2 is incorrect.

The annual mean temperature of the world is known to have increased by 1.1 degree Celsius from the average of
the 1850-1900 period. But this increase, as can be expected, is not uniform. It varies in different regions and also
at different times of the year.

Temperature rise over land is much higher than over oceans. The increase in temperatures is known to be more
prominent in the higher altitudes, near the polar regions, than near the equator. This is attributable to a
complex set of atmospheric phenomena, including heat transfers from the tropics to the poles through prevailing
systems of air circulation. India happens to be in the tropical region, quite close to the equator.

The polar regions, particularly the Arctic, have seen significantly greater warming. The IPCC report says the Arctic
region has warmed at least twice as much as the world average. Its current annual mean temperatures are about
2 degrees Celsius higher than pre-industrial times. Some other studies suggest the Arctic could be warming even
faster.

Another prominent cause is what is known as the albedo effect, or how much sunlight a surface reflects. The ice
cover in the Arctic is melting, because of which more land or water is getting exposed to the Sun. Ice traps the
least amount of heat and reflects most of the solar radiation when compared with land or water.

The warming trends over the Indian region are very different. An assessment of climate change over the Indian
subcontinent, published by the Ministry of Earth Sciences in 2020, said annual mean temperatures had risen by
0.7 degree Celsius from 1900. This is significantly lower than the 1.59 degree Celsius rise for land temperatures
across the world.

6) Consider the following pairs.


1. Kakrapar Atomic power plant: Rajasthan
2. Tarapur Atomic power plant: Gujarat
3. Narora Atomic power plant: Uttar Pradesh
4. Kaiga Atomic power plant: Karnataka
How many of the above pairs is/are correctly matched?
a) Only one
b) Only two
c) Only three
d) All four

Solution: b)

Statement 3 and 4 is correct.

www.insightsonindia.com 27
INSTA CURRENT AFFAIRS QUIZ

7) Consider the following statements.


1. Anabatic Winds are downslope winds created when the mountain surface is colder than the
surrounding air and create a downslope wind.
2. Katabatic Winds are upslope winds driven by warmer surface temperatures on a mountain slope than
the surrounding air column.
Which of the above statements is/are incorrect?
a) 1 only
b) 2 only
c) Both 1 and 2
d) Neither 1 nor 2

Solution: c)

Anabatic Winds – These Winds are upslope winds driven by warmer surface temperatures on a mountain slope
than the surrounding air column.
www.insightsonindia.com 28
INSTA CURRENT AFFAIRS QUIZ
Katabatic Winds – Katabatic winds are downslope winds created when the mountain surface is colder than the
surrounding air and create a downslope wind.

Source

8) Consider the following statements.


1. Earthquakes can result in Tsunamis if a large amount of water gets displaced.
2. All earthquakes do not result in Tsunamis.
Which of the above statements is/are incorrect?
a) 1 only
b) 2 only
c) Both 1 and 2
d) Neither 1 nor 2

Solution: d)

Earthquakes can result in Tsunamis if a large amount of water gets displaced.

Tectonic events with a strong vertical slip element are more likely to raise or lower the water column compared
to horizontal slip elements”. “Thus, knowing the slip type at the early stages of the assessment can reduce false
alarms and enhance the reliability of the warning systems.

Relying on seismographs and buoys attached by bottom-pressure sensors is not always the right method, as all
earthquakes do not result in Tsunamis, and the buoys can detect Tsunamis only when one passes them, leaving
less time to react and evacuate. In such cases, time is of the essence.

9) Consider the following statements regarding Atmospheric rivers.


1. Atmospheric rivers are short and wide regions of the atmosphere that transport water vapour
from one region to the neighbouring region.
2. When atmospheric rivers make landfall, they can contribute significantly to precipitation.
3. Events like the 2018 Kerala floods, the 2022 Pakistan floods, and the lower Mississippi River floods
were linked to atmospheric rivers.
How many of the above statements is/are correct?
a) Only one
b) Only two
c) All three
d) None

Solution: b)

www.insightsonindia.com 29
INSTA CURRENT AFFAIRS QUIZ

Statement 1 is incorrect.

A recent study published in the American Meteorological Society’s Journal of Hydrometeorology reveals
that peninsular river basins in India, including Narmada, Mahanadi, and Godavari, face a higher likelihood of
widespread flooding compared to transboundary rivers like Ganga and Brahmaputra.

Drivers of widespread flooding are associated with atmospheric rivers, large atmospheric circulations carrying
moisture from the tropics to extratropic. Events like the 2018 Kerala floods, the 2022 Pakistan floods, and the
lower Mississippi River floods in 2008, 2011, and 2015–19 were linked to atmospheric rivers.

What are Atmospheric rivers?


They are long, narrow regions of the atmosphere that transport water vapour from the tropics to the poles. They
can be 250–375 miles wide and over 1,000 miles long. When atmospheric rivers make landfall, they can
contribute significantly to precipitation.

Source

10) Consider the following statements regarding Himalayan River System.


1. The Rivers have large basins and catchment areas.
2. They flow through deep V-shaped valleys called gorges.
3. They are the examples of consequent drainage system.
4. The Rivers flow only in rainy season.
How many of the above statements is/are correct?
a) Only one
b) Only two
c) Only three
d) All four

Solution: b)

Only statement 1 and 2 is correct.

www.insightsonindia.com 30
INSTA CURRENT AFFAIRS QUIZ

11) Consider the following statements regarding Iridescent clouds.


1. Iridescent clouds occurs as sunlight diffracts around tiny ice crystals within the clouds, creating an
effect similar to a thin layer of oil on water.
2. Nacreous clouds are usually seen in UK as the cold air from the polar vortex always hovers over the
region.
3. The clouds are known for their pastel light reflections during sunset and sunrise.
How many of the above statements is/are correct?
a) Only one
b) Only two
c) All three
d) None

Solution: b)

Statement 2 is incorrect.

www.insightsonindia.com 31
INSTA CURRENT AFFAIRS QUIZ
• Rare iridescent clouds, known as nacreous clouds, were spotted over parts of Scotland, northern England, and
the West Midlands recently.
• The clouds, resembling mother of pearl, are known for their pastel light reflections during sunset and
sunrise.
• This phenomenon occurs as sunlight diffracts around tiny ice crystals within the clouds, creating an effect
similar to a thin layer of oil on water.
• Nacreous clouds are rare in the UK and are usually seen when cold air from the polar vortex temporarily
hovers over the region.

Source

12) Consider the following statements.


1. Pong Dam Wildlife Sanctuary known for its diverse flora and fauna is located in Gujarat.
2. Pong Dam Lake is a manmade reservoir created with construction of Pong Dam.
3. Pong Dam Lake is designated as a Ramsar site.
How many of the above statements is/are correct?
a) Only one
b) Only two
c) All three
d) None

Solution: b)

Statement 1 is incorrect.

About the Pong Dam Wildlife Sanctuary


• It is located in the Kangra district of Himachal Pradesh, is a significant protected area known for its diverse
flora and fauna.
• Pong Dam Lake (Maharana Pratap Sagar) is a manmade reservoir created in 1975 with construction of Pong
Dam on Beas River. It is also known as Pong reservoir or Pong Dam.
• In 1983, reservoir was declared as Wildlife Sanctuary.
• Pong Dam Lake was designated a Ramsar site in 2002. There is a Vulture café in Pong wherein cattle carcass
is dumped for Vultures.

Source

13) Consider the following statements.


1. Glaciers are the largest source of freshwater in the Himalayan ecosystem.
2. A glacial lake outburst flood (GLOF) is caused by the failure of a dam containing a glacial lake.
3. There is considerable shrinking of glacial lakes across the Himalayas, in recent decade.
How many of the above statements is/are correct?
a) Only one
b) Only two
www.insightsonindia.com 32
INSTA CURRENT AFFAIRS QUIZ
c) All three
d) None

Solution: b)

Statement 3 is incorrect.

Glaciers are the largest source of freshwater outside of the polar regions. Glaciers and snow melt in the
Himalayan ecosystem are the source of water for several rivers across the subcontinent, and are responsible for
maintaining the perennial supply of water in the river systems like the Indus, Ganges, and Brahmaputra to over a
billion people.

Some models predict that an increase in global temperatures by 2°C from 1850 by 2070 would result in 45% of
the medium and large glaciers (10 sq km or more) disappearing completely. Nearly 70% smaller glaciers are likely
to melt away. Shrinking glaciers have led to the formation of a large number of glacial lakes all across the
Himalayas. Many of these high-altitude lakes are potentially dangerous, because of their potential to cause flash
floods in the event of a breach.

A glacial lake outburst flood (GLOF) is a type of outburst flood caused by the failure of a dam containing a
glacial lake.

14) Consider the following statements regarding Ionosphere.


1. The ionosphere is a layer of the Earth’s atmosphere that is made up of charged atoms, molecules, and
electrons.
2. It stretches from 50 kilometres to over 1,000 kilometres above the Earth.
3. It is located below the Mesosphere.
4. Satellite and communication systems cannot function in this layer.
How many of the above statements is/are correct?
a) Only one
b) Only two
c) Only three
d) All four

Solution: b)

Only statement 1 and 2 is correct.

Scientists from the Indian Institute of Geomagnetism (IIG) have conducted decade-long ionospheric
observations at Bharati station in Antarctica, revealing significant seasonal variations in ionospheric density.

The researchers found the peak ionospheric density near local noon during the winter months, attributing it
to particle precipitation and the transportation of convectional plasma from high latitudes.
These long-term studies aim to enhance understanding of ionospheric effects on satellite-based navigation and
communication systems.

The ionosphere is a layer of the Earth’s atmosphere that is made up of charged atoms, molecules, and
electrons. It stretches from 50 kilometres (30 miles) to over 1,000 kilometres (600 miles) above the Earth.

www.insightsonindia.com 33
INSTA CURRENT AFFAIRS QUIZ

4. Government Schemes and Programmes


1) Consider the following statements regarding Moving Off Information Systems (MOIS).
1. Moving Off Information Systems (MOIS) is an advanced driver assistance system that detects and
informs the driver of the presence of pedestrians and cyclists in the close-proximity forward blind spot of the
vehicle, aiming to enhance road safety.
2. The Government has made it mandatory to install Moving Off Information Systems in all vehicles
carrying passengers or goods.
Which of the above statements is/are correct?
a) 1 only
b) 2 only
c) Both 1 and 2
d) Neither 1 nor 2

Solution: a)

The government’s technical committees, including the Central Motor Vehicle Rules-Technical Standing
Committee (CMVR-TSC) and the Automotive Industry Standards Committee (AISC), are working on
formulating standards for Moving Off Information Systems (MOIS) in vehicles.

What is MOIS?
• Moving Off Information Systems (MOIS) is an advanced driver assistance system designed to aid drivers
during low-speed moving-off manoeuvres, particularly in situations involving vehicles carrying passengers or
goods.
• The system detects and informs the driver of the presence of pedestrians and cyclists in the close-proximity
forward blind spot of the vehicle, aiming to enhance road safety.
• Additionally, the National Highways Authority of India (NHAI) has developed a Computer Aided Dispatch
System and NHAI ERS Mobile Application to expedite emergency response calls, aligning with the broader
focus on improving traffic safety as per the Motor Vehicles Act, 1988.

Source

2) Consider the following statements regarding Dare to Dream (D2D) scheme.


1. The Dare to Dream (D2D) scheme is a contest that provides a unique opportunity for start-ups,
entrepreneurs, and individuals to enhance India’s defence and aerospace capabilities.
www.insightsonindia.com 34
INSTA CURRENT AFFAIRS QUIZ
2. It is an annual initiative by the Defence Research and Development Organisation (DRDO).
3. It is initiated in memory of former President Dr APJ Abdul Kalam.
How many of the above statements is/are correct?
a) Only one
b) Only two
c) All three
d) None

Solution: c)

The Dare to Dream Contest, initiated in memory of former President Dr APJ Abdul Kalam, is an annual initiative
by the Defence Research and Development Organisation (DRDO).
Launched in 2019, the contest provides a unique opportunity for start-ups, innovators, entrepreneurs, and
individuals above 18 years old to address key challenges in emerging technologies, ultimately contributing to
enhancing India’s defence and aerospace capabilities.

Source

3) PM-AASHA Scheme is an umbrella scheme aimed to-


a) Enhance access to secondary education in India and improve its quality.
b) Ensure Minimum Support Price to the farmers.
c) Provide health insurance scheme for low-income families.
d) Provide volunteers for campaigning healthcare schemes.

Solution: b)

The Prime Minister's Farmer Income Protection Scheme also known as PM AASHA is an umbrella scheme of
the Government of India to ensure a price policy such as a minimum support price are fulfilled. It includes the
former Price Support Scheme (PSS), a Price Deficiency Payment Scheme (PDPS) and a Private Procurement &
Stockist Scheme (PPPS).

4) Consider the following statements regarding Mission on Advanced and High Impact Research
1. It is jointly funded by DRDO and the World Bank.
2. It aims to achieve Net Zero emissions along with promoting the UN Sustainable Development goals.
3. It will collaborate with IITs and IIMs to identify emerging technologies in the power sector.
4. It will facilitate commercialisation of indigenous Technologies developed by Indian startups.
How many of the above statements are correct?
a) Only one
b) Only two
c) Only three
d) All four

Solution: c)

Statement 1 is incorrect.

• The Mission on Advanced and High Impact Research has been launched jointly by the Ministry of Power
and Ministry of New and Renewable Energy
• It is aimed to identify emerging technologies and areas of future relevance for the Global power sector and to
take up indigenous end to end development of relevant Technologies.
• the mission will run for an initial period of 5 years from 2023-24 to 2027-28.
• It will be funded by Ministry of Power, Ministry of New and Renewable Energy and Central Public Sector
Enterprises under these ministries
• additional funding if required, will be provided from budgetary sources of the central government.

www.insightsonindia.com 35
INSTA CURRENT AFFAIRS QUIZ
• The mission will serve as a catalyst for National priorities such as achieving Net Zero emissions and
promoting initiatives like Make in India and Start Up India
• it will also contribute towards achieving United Nations’ Sustainable Development goals.
• The project will support pilot projects of indigenous Technologies developed especially by Indian startups
and facilitate their commercialisation.
• MAHIR will work with premier institutions such as IITs, IIMs, NITs, IISR and universities on one hand and public
and private power sector startups and established industries with the government acting as an enabler for
creating an innovation ecosystem in the country.

5) Consider the following statements.


1. People with blood disorders like thalassemia, haemophilia, and sickle cell disease are eligible for job
reservations in government roles under the Persons with Disabilities category.
2. As per rules regarding Reservations for disabled people, 4% reservation in government jobs is available
for people with disabilities.
3. The Rights of Persons with Disabilities Act includes Intellectual disability and Mental illnesses as
benchmark disabilities.
How many of the above statements is/are correct?
a) Only one
b) Only two
c) All three
d) None

Solution: b)

Statement 1 is incorrect.

People with blood disorders like thalassemia, haemophilia, and sickle cell disease are not eligible for job
reservations in government roles under the Persons with Disabilities category.
The three diseases were included in the list of disabilities under the Rights of Persons with Disabilities Act of 2016,
but they are excluded from job reservation benefits.

Rules regarding Reservations for disabled people:


• 4% reservation in government jobs is available for people with disabilities
• Every government establishment must reserve 1% of the total number of vacancies for people with
benchmark disabilities.

The RPwD Act defines benchmark disabilities as:


• Autism
• Intellectual disability
• Specific learning disability
• Mental illnesses

The RPwD Act also defines disabilities as:


• Blindness and low vision
• Deaf and hard of hearing
• Locomotor disability, including cerebral palsy, leprosy, dwarfism, acid attack victims, and muscular dystrophy

Source

6) Consider the following statements regarding ONGC.


1. It is a Navratna company contributing around 71%to the Indian domestic production.
2. ONGC has a unique distinction of being a company with in-house service capabilities in all areas of
Exploration and Production of oil & gas.
Which of the above statements is/are correct?

www.insightsonindia.com 36
INSTA CURRENT AFFAIRS QUIZ
a) 1 only
b) 2 only
c) Both 1 and 2
d) Neither 1 nor 2

Solution: b)

Maharatna ONGC is the largest crude oil and natural gas Company in India, contributing around 71 per cent to
Indian domestic production. Crude oil is the raw material used by downstream companies like IOC, BPCL, HPCL
and MRPL (Last two are subsidiaries of ONGC) to produce petroleum products like Petrol, Diesel, Kerosene,
Naphtha, and Cooking Gas LPG.
ONGC has a unique distinction of being a company with in-house service capabilities in all areas of Exploration
and Production of oil & gas and related oil-field services.

7) Consider the following statements regarding the recently launched Project Prayas.
1. The project aims to facilitate safe, orderly, and regular migration for Indian workers and students.
2. It consolidates state-level initiatives and focuses on improving coordination between state and central
governments.
3. It was launched by the Union Ministry of Home Affairs.
How many of the above statements is/are correct?
a) Only one
b) Only two
c) All three
d) None

Solution: b)

Statement 3 is incorrect.

The International Organization for Migration (IOM) has launched Project PRAYAS (Promoting Regular &
Assisted Migration for Youth and Skilled Professionals) in collaboration with the Ministry of External Affairs.

About Project Prayas:


• The project aims to facilitate safe, orderly, and regular migration for Indian workers and students.
• It consolidates state-level initiatives and focuses on improving coordination between state and central
governments.
• The project involves studying migration trends, understanding migrants’ needs, and launching initiatives to
raise awareness about safe migration processes, with the goal of strengthening international migration
governance.

Source

8) India ensures the welfare of its diaspora through which of the following initiatives?
1. Pre-departure orientation training
2. Indian Community Welfare Fund
3. MADAD Portal
4. Know India Programme
How many of the above options is/are correct?
a) Only one
b) Only two
c) Only three
d) All four

Solution: d)

www.insightsonindia.com 37
INSTA CURRENT AFFAIRS QUIZ
India has a vast diaspora exceeding 32 million globally as of December 2021. It is the world’s top recipient of
remittances. India ensures the welfare of its diaspora through initiatives like pre-departure orientation training,
the Indian Community Welfare Fund (ICWF) for assistance in distress, the MADAD Portal for grievance redressal,
and the Know India Programme, exposing the young diaspora to contemporary India.

9) Consider the following statements regarding Palna Scheme.


1. Palna Scheme is a centrally sponsored initiative by the Ministry of Women and Child Development.
2. The scheme is part of Samarthya which is a sub-component under Mission Shakti.
3. The scheme aims to address the urgent need for quality day care facilities for the children of working
women.
4. The scheme initially focus only in rural areas.
How many of the above statements is/are correct?
a) Only one
b) Only two
c) Only three
d) All four

Solution: c)

Statement 4 is incorrect.

10) Which of the following statements is correct regarding the recently launched “MedTech Mitra” initiative?
a) Provide financial assistance to poor patients suffering from major life-threatening diseases
b) Delivering critical care medicines through a drone
c) It aims to empower innovators and advance healthcare solutions in the country.
www.insightsonindia.com 38
INSTA CURRENT AFFAIRS QUIZ
d) Connecting all health and wellness centres using the hub-and-spoke model

Solution: c)

The Union Minister of Health and Family Welfare in India, has launched the “MedTech Mitra” initiative to
empower innovators and advance healthcare solutions in the country.
The initiative aims to support and collaborate with technology innovators and startups in the medical technology
sector.
The goal is to harness innovation to enhance healthcare delivery and address healthcare challenges.

Source

11) The Pradhan Mantri JI-VAN Yojana aims to


a) Set up third generation (3G) ethanol projects
b) Promote the use of Compressed BioGas (CBG) as an alternative green transportation fuel
c) Provide financial support to integrated bio-ethanol projects
d) Convert cattle dung and other organic waste into compost, biogas, and biofuels

Solution: c)

The Pradhan Mantri JI-VAN Yojana provides financial support to integrated bio-ethanol projects to set up
second generation (2G) ethanol projects.

5. Science and Technology


1) Which of the following are the examples of Oxidation Reaction?
1. The taste of butter changes if left for a longer time
2. Wine gets sour with time
3. A reddish brown powder gets coated on articles made of iron
How many of the above statements is/are correct?
a) Only one
b) Only two
c) All three
d) None

Solution: c)

The term oxidation was originally used to describe reactions in which an element combines with oxygen. The
iron metal is oxidized to form the iron oxide known as rust.
Electrochemical reactions are great examples of oxidation reactions.
All the above are the examples of Oxidation Reaction.

2) Consider the following statements regarding Attention-deficit hyperactivity disorder (ADHD).


1. ADHD is the neurodevelopmental disorders of childhood diagnosed at an early age.
2. Children with ADHD might have trouble paying attention or controlling impulsive behaviours.
3. ADHD does not last into adulthood.
How many of the above statements is/are correct?
a) Only one
b) Only two
c) All three
d) None

www.insightsonindia.com 39
INSTA CURRENT AFFAIRS QUIZ

Solution: b)

Statement 3 is incorrect.

ADHD is one the most common neurodevelopmental disorders of childhood, usually diagnosed at an early age
and lasting into adulthood. According to the US Centres for Disease Control and Prevention (CDC), children with
ADHD might have trouble paying attention or controlling impulsive behaviours, and can be overly active.
Many adults remain undiagnosed, and symptoms can cause difficulty at work, home or relationships. The CDC
states that symptoms can appear different at older ages, for example, hyperactivity can appear as extreme
restlessness.

For people diagnosed with ADHD, Adderall can help improve focus, concentration, impulse control and
hyperactivity, because of its effects on the central nervous system.

3) Variola virus is commonly associated with which of the following disease?


a) Measles
b) Polio
c) Smallpox
d) Hepatitis B

Solution: c)

The history of smallpox holds a special place in the field of epidemiology. One of the deadliest diseases known to
mankind, it is also the only one to have been eradicated by vaccination. However, the origins of smallpox have
always been unclear. Until now, the earliest confirmed case of the disease was found in the mummified remains
of a 17th-century Lithuanian child, even though written records suggest that it is much older. A new study carried
out by an international team of researchers provides fresh insight into the origins of the disease caused by the
Variola virus (VARV), suggesting that the smallpox was in existence as early as during the Viking age in the 8th
century CE.

4) Consider the following statements regarding AKTOCYTE, that was recently in news
1. These are tablets that aim to enhance the quality of life for cancer patients undergoing radiotherapy.
2. It has been developed by Department of Atomic Energy (DAE).
3. It has received approval from the Food Safety and Standards Authority of India (FSSAI), ensuring its
safety and compliance with quality standards.
How many of the above statements is/are correct?
a) Only one
b) Only two
c) All three
d) None

Solution: c)

• Scientists from the Department of Atomic Energy (DAE) and M/s. IDRS Labs Pvt. Ltd. in Bengaluru have
collaboratively developed a breakthrough nutraceutical named AKTOCYTE tablets.
• These tablets aim to enhance the quality of life for cancer patients undergoing radiotherapy by minimizing
the side effects associated with the treatment.
• AKTOCYTE tablets have shown remarkable results, particularly in pelvic cancer
patients undergoing radiotherapy-induced Cystitis, leading to extraordinary recovery and eliminating the
need for surgical interventions like the removal of the urinary bladder.
• AKTOCYTE has received approval from the Food Safety and Standards Authority of India (FSSAI), ensuring
its safety and compliance with quality standards.

www.insightsonindia.com 40
INSTA CURRENT AFFAIRS QUIZ
Source

5) Consider the following statements regarding Dark fibre.


1. Dark fibre is unused fibre-optic cable and infrastructure that is available for purchase or rent from
network service providers.
2. Dark fibre is inactive because no information is passed through it, so no light pulse is transmitted.
3. SEBI bans the usage of Dark fibre in India.
How many of the above statements is/are correct?
a) Only one
b) Only two
c) All three
d) None

Solution: b)

Statement 3 is incorrect.

The Securities Appellate Tribunal (SAT) has overturned the Securities and Exchange Board of India’s (Sebi) fine on
the National Stock Exchange (NSE) in the dark-fibre case.
The investigation focused on potential violations of regulations and circulars related to fraudulent and unfair
trade practices.

About Dark Fiber:


• Dark fibre, also known as unlit fibre, is unused fibre-optic cable and infrastructure. It’s available
for purchase or rent from network service providers.
• Dark fibre is used for private networking, internet access, and internet infrastructure networking. It’s typically
used for telecom and network communications.
• Dark fibre is different from “lit fibre” which refers to cables that are actively being used. Dark fibre is inactive
because no information is passed through it, so no light pulse is transmitted.

Source

6) Consider the following statements.


1. Nuclear Power Corporation of India Limited (NPCIL) is a public sector undertaking under the Ministry of
Power.
2. The pressurized heavy water reactors (PHWRs) utilize natural uranium as fuel and heavy water
as coolant and moderator.
3. Kakrapar Atomic Power facility houses the largest indigenous nuclear power reactors built by
the Nuclear Power Corporation of India Limited (NPCIL).
How many of the above statements is/are correct?
a) Only one
b) Only two
c) All three
d) None

Solution: b)

Statement 1 is incorrect.

• The fourth unit of the Kakrapar Atomic Power Project (KAPP-4) in Gujarat, with a capacity of 700 MWe,
achieved criticality recently, marking the initiation of a controlled fission chain reaction.
• The Kakrapar facility, houses the largest indigenous nuclear power reactors built by the Nuclear Power
Corporation of India Limited (NPCIL).
• These pressurized heavy water reactors (PHWRs) utilize natural uranium as fuel and heavy water as coolant
and moderator.
www.insightsonindia.com 41
INSTA CURRENT AFFAIRS QUIZ
• The Kakrapar Atomic Power Station includes two operating PHWRs with a capacity of 220 MWe each, known
as KAPS-1 and -2.
• The NPCIL, a public sector undertaking of the Department of Atomic Energy, already operates indigenous
PHWRs at other facilities.

Source

7) Consider the following statements regarding Juice space mission, recently seen in news.
1. Juice is an interplanetary spacecraft launched by NASA.
2. The mission aims to carry out a detailed exploration of Jupiter and its icy moons.
3. Juice will be the first spacecraft to examine Jupiter.
How many of the above statements is/are correct?
a) Only one
b) Only two
c) All three
d) None

Solution: a)

Only Statement 2 is correct.

The European Space Agency (ESA) launched the Jupiter Icy Moons Explorer, or Juice, mission from its spaceport
in French Guiana on an Ariane 5 launcher. Planned to reach Jupiter in 2031, the mission aims to carry out a
detailed exploration of the Solar System’s largest planet and its icy moons, which potentially have habitable
environments.

Only two other spacecraft have ever examined Jupiter: the Galileo probe, which orbited the gas giant between
1995 and 2003, and Juno, which has been circling the planet since 2016. Notably, by the time Juice reaches
Jupiter, another spacecraft, NASA’s Europa Clipper, would already be orbiting the planet — scheduled to be
launched in October this year, Europa Clipper would arrive at Jupiter in 2030 and aims to study its Europa moon.

According to ESA’s website, the Juice “will make detailed observations of the giant gas planet and its three large
ocean-bearing moons — Ganymede, Callisto and Europa”, by using remote sensing, geophysical and in situ
instruments.
Scientists for quite some time have known that these three moons of Jupiter possess icy crusts, which they
believe contain oceans of liquid water underneath, making them potentially habitable. Juice will help probe these
water bodies by creating detailed maps of the moons’ surfaces and enable the scientists, for the first time, to look
beneath them.

8) Consider the following statements regarding narco analysis test.


1. In a narco analysis test, a drug called sodium pentothal is injected into the body of the accused.
2. During the test, the person enters into a hypnotic or sedated state in which their imagination is
neutralised.
3. The procedure is similar to polygraph test, which is also a lie-detector test.
How many of the above statements are correct?
a) Only one
b) Only two
c) All three
d) None

Solution: b)

Statement 3 is incorrect.

www.insightsonindia.com 42
INSTA CURRENT AFFAIRS QUIZ
In a ‘narco’ or narco analysis test, a drug called sodium pentothal is injected into the body of the accused,
which transports them to a hypnotic or sedated state in which their imagination is neutralised. In this hypnotic
state, the accused is understood as being incapable of lying and is expected to divulge information that is true.

Because the drug is believed to weaken the subject’s resolve to lie, it is sometimes referred to as a “truth serum”.

However, narco tests must not be confused with polygraph tests, which, although having the same truth-
decoding motive, work differently.

9) Consider the following pairs regarding the Diseases and their impact.
1. Thalassemia: Reduced ability to produce normal haemoglobin
2. Haemophilia: Prolonged bleeding and difficulty in clotting
3. Sickle Cell Disease: Altered red blood cell shape
How many of the above pairs is/are correct?
a) Only one
b) Only two
c) All three
d) None

Solution: c)

Condition Nature Impact


Reduced ability to produce normal
Thalassemia Blood disorder affecting haemoglobin
haemoglobin
Bleeding disorder, deficiency of
Haemophilia Prolonged bleeding, difficulty in clotting
clotting factor
Sickle Cell Genetic blood disorder, abnormal Altered red blood cell shape, circulation
Disease hemoglobin issues

10) Consider the following statements regarding Zero Trust Authentication (ZTA).
1. Zero Trust Authentication (ZTA) is a security framework that requires users to be authenticated and
authorized before being granted access to applications and data.
2. It includes two-factor authentication with options like facial recognition or biometrics.
3. Here the log-in and log-out times are recorded and monitored.
How many of the above statements is/are correct?
a) Only one
b) Only two
c) All three
d) None

Solution: c)

Zero Trust Authentication (ZTA) is a security framework that requires users to be authenticated and
authorized before being granted access to applications and data.

• In response to the increasing threat of cyberattacks, the Indian government has established a secure e-mail
system for 10,000 users in critical ministries and departments.
• The e-mail system operates on Zero Trust Authentication (ZTA) and has been developed by the National
Informatics Centre (NIC).
• The security measures include two-factor authentication with options like facial recognition or
biometrics, and log-in and log-out times are recorded and monitored.

Source

www.insightsonindia.com 43
INSTA CURRENT AFFAIRS QUIZ

11) Consider the following statements regarding touchscreens.


1. Capacitive touchscreens have two conductive sheets separated by a small gap.
2. Resistive touchscreens operate by detecting changes in electric fields when a finger touches the screen.
3. Capacitive touchscreens are increasingly used in consumer electronics due to their superior image
clarity, sensitivity, and durability.
How many of the above statements is/are correct?
a) Only one
b) Only two
c) All three
d) None

Solution: a)

Only Statement 3 is correct.

There are two main types of touchscreens, capacitive and resistive.


• Capacitive touchscreens, used in smartphones, operate by detecting changes in electric fields when a finger
touches the screen.
• Resistive touchscreens, with two conductive sheets separated by a small gap, function based on the change
in resistance when touched.

The rising dominance of capacitive touchscreens in consumer electronics since 2007 due to their superior image
clarity, sensitivity, and durability compared to resistive touchscreens.

Source

12) Consider the following statements regarding TEMPO satellite.


1. It is an Earth-observation satellite in geostationary orbit.
2. It is designed to provide hourly measurements of pollutants over Indian Sub-continent.
3. It is built by ISRO in collaboration with NASA.
How many of the above statements is/are correct?
a) Only one
b) Only two
c) All three
d) None

Solution: a)

Only Statement 1 is correct.

• NASA’s TEMPO satellite, the Tropospheric Emissions: Monitoring of Pollution sensor, is a ground breaking
Earth-observation satellite in geostationary orbit designed to provide hourly measurements of pollutants over
North America.
• TEMPO covers a vast area from Canada’s oil sands to the Yucatán Peninsula and spans across the Atlantic to
the Pacific Ocean, enhancing its ability to monitor air pollution comprehensively.

Source

13) Consider the following statements.


1. Diesel as a fuel contains less carbon per litre than petrol.
2. Diesel engines uses greater amount of fuel to produce the same level of performance as a petrol
engine.
3. Diesel was promoted as a green-friendly auto fuel as part of the European Union’s response to the
Kyoto Protocol to reduce carbon-dioxide emissions.
www.insightsonindia.com 44
INSTA CURRENT AFFAIRS QUIZ
How many of the above statements are correct?
a) Only one
b) Only two
c) All three
d) None

Solution: a)

Only statement 3 is correct.

Diesel was promoted as a green-friendly auto fuel as part of the European Union’s response to the Kyoto
Protocol (1997) to reduce carbon-dioxide emissions. While diesel as a fuel contains a little more carbon per litre
than petrol, diesel engines are more “lean-burn”, which means they use a smaller amount of fuel to produce the
same level of performance as a petrol engine.

14) Consider the following statements regarding UPI tap and pay.
1. It utilizes near-field communication (NFC) technology to capture a payee’s UPI ID or Virtual Payment
Address (VPA).
2. Transactions up to ₹5000 can be done without any pin number using UPI tap and pay.
3. Near-field communication uses a phone’s camera to enable communication.
How many of the above statements is/are correct?
a) Only one
b) Only two
c) All three
d) None

Solution: a)

Only Statement 1 is correct.

The National Payments Corporation of India (NPCI) has initiated the deployment of ‘UPI Tap and Pay’ across
digital payment providers.

Features of UPI tap and pay:


• It utilizes near-field communication (NFC) technology to capture a payee’s UPI ID or Virtual Payment Address
(VPA), eliminating the need for a camera.
• The feature can only be accessed on phones or devices with NFC capability.
• Users need to locate the ‘Tap & Pay’ button on their UPI app, and the transaction is completed by tapping
the device on the UPI Smart Tag/Smart QR.
• Transactions up to ₹500 are processed through UPI LITE, while those exceeding ₹500 require a UPI PIN.
Near-field communication (NFC) is a short-range wireless technology that allows two devices to communicate
when they are within 4 centimetres of each other. NFC uses magnetic field induction to enable communication.

Source

15) Consider the following statements regarding Formaldehyde.


1. Formaldehyde is a colourless, flammable gas with a strong odour.
2. It is used in Textiles, Disinfectants, Building materials and Automobile parts.
3. It is classified as safe and non-carcinogenic without any adverse health effects.
How many of the above statements is/are correct?
a) Only one
b) Only two
c) All three
d) None

www.insightsonindia.com 45
INSTA CURRENT AFFAIRS QUIZ
Solution: b)

Statement 3 is incorrect.

About Formaldehyde
• It is a colourless, flammable gas with a strong odour. It is classified as a human carcinogen and is linked to
long-term adverse health effects and cancer.
• Formaldehyde is used in many industries, including:
• Glues and resins, Dyes, Textiles, Disinfectants, Building materials, Automobile parts, Embalming,
Laboratories.

Source

6. International Relations and Organisations


1) Carbon Border Adjustment Mechanism (CBAM), sometimes seen in news is the initiative of
a) G20 Grouping
b) OECD
c) European Union (EU)
d) United States

Solution: c)

The EU in 2021, came up with a proposal for a Carbon Border Adjustment Mechanism (CBAM).
The CBAM plans to impose a tariff on a set of carbon-intensive imports, which will have to be paid by EU
importers and companies who export such goods to EU countries.
With the CBAM, the EU also wants to create a level-playing field for business in the bloc with those outside by
making equal the price for the carbon content of goods regardless of where they are made.

2) Consider the following statements regarding Internal Complaints Committee (ICC).


1. The constitution of Internal Complaints Committee (ICC) is mandated by law under the 2013 Prevention
of Sexual Harassment (PoSH) Act.
2. As per law, the Internal Complaints Committee needs to have a minimum of four members, with at
least half of them women.
3. A functional Internal Complaints Committee is one of the key conditions set up by the Ministry of
Sports to grant annual recognition to the national sports federations.
How many of the above statements is/are correct?
a) Only one
b) Only two
c) All three
d) None

Solution: c)

Internal Complaints Committee (ICC), as mandated by law under the 2013 Prevention of Sexual Harassment
(PoSH) Act.

The ICC was designed to be the first port of call for any grievance under the PoSH Act, a key element needed to
create a safe workplace environment for women.

As per the law, it needs to have a minimum of four members – at least half of them women – of whom one shall
be an external member, preferably from an NGO or an association that works for women’s empowerment or a

www.insightsonindia.com 46
INSTA CURRENT AFFAIRS QUIZ
person familiar with issues related to sexual harassment, like a lawyer. In fact, a functional ICC is one of the key
conditions set up by the Ministry of Sports to grant annual recognition to the federations.

3) Consider the following statements regarding Tax Inspectors Without Borders (TIWB) program.
1. Tax Inspectors Without Borders (TIWB) program, is a collaborative effort between World Bank and the
Organisation for Economic Cooperation and Development (OECD).
2. India has been selected as the Partner Administration.
3. It will facilitate the transfer of tax audit knowledge and skills to developing country tax administrations.
How many of the above statements is/are correct?
a) Only one
b) Only two
c) All three
d) None

Solution: b)

Statement 1 is incorrect.

• Tax Inspectors Without Borders (TIWB) program, a collaborative effort between the United Nations
Development Programme (UNDP) and the Organisation for Economic Cooperation and Development
(OECD), was recently launched in Saint Lucia.
• India has been selected as the Partner Administration and will contribute Tax Experts to the 12-18-month
initiative. This marks the seventh TIWB program supported by India through the provision of Tax Experts.
• Aim: The program’s focus is on the effective use of automatic exchange of information under the Common
Reporting Standard (CRS) framework. TIWB will facilitate the transfer of tax audit knowledge and skills to
developing country tax administrations using a practical, “learning by doing” approach.

Source

4) Consider the following statements.


1. India has signed the Svalbard Treaty, which is regarding the country’s engagement with the Arctic
region.
2. Maitri is India’s Arctic Research Station.
3. As per present International law, India is not allowed to operate its Arctic research bases during winter.
How many of the above statements is/are correct?
a) Only one
b) Only two
c) All three
d) None

Solution: a)

Only Statement 1 is correct.

India’s engagement with the Arctic began when it signed the Svalbard Treaty in February 1920 in Paris between
Norway, the US, Denmark, France, Italy, Japan, the Netherlands, Great Britain, and Ireland, and the British
overseas Dominions and Sweden concerning Spitsbergen. Ever since then, India has been closely monitoring all
the developments in the Arctic region.

India has launched its first winter expedition to Himadri, its Arctic Research Station at Ny-Ålesund in the
Norwegian archipelago of Svalbard. The station, which was previously operational for about 180 days
annually, will now remain operational throughout the year. This move makes India one of the few countries to
operate its Arctic research bases during winter.

Source
www.insightsonindia.com 47
INSTA CURRENT AFFAIRS QUIZ

5) Consider the following statements regarding United Nations Security Council (UNSC).
1. The UN Security Council is the only UN body with the authority to issue binding resolutions.
2. UNSC can take action in situations threatening international peace and security.
3. All decisions of the UNSC must be approved by the UN General Assembly to be enforced.
How many of the above statements is/are correct?
a) Only one
b) Only two
c) All three
d) None

Solution: b)

Statement 3 is incorrect.

Decisions taken under Chapter VII, such as economic sanctions or other sections like international security, are
binding on UN members; the Security Council is the only UN body with the authority to issue binding
resolutions. The UNGA can only make recommendations in this regard.

Decisions that are vetoed by the P-5 cannot be submitted for the review of the UNGA. Also, the UNSC is the
authority under the UN charter to take action in situations threatening international peace and security.

6) Consider the following statements regarding EU’s Generalised Scheme of Preferences Plus (GSP+).
1. It gives developing countries a special incentive to pursue sustainable development and good
governance.
2. The EU cuts its import duties to zero on more than two thirds of the tariff lines of the exports from the
developing countries.
Which of the above statements is/are correct?
a) 1 only
b) 2 only
c) Both 1 and 2
d) Neither 1 nor 2

Solution: c)

(The European Commission’s website describes the EU’s Generalised Scheme of Preferences Plus (GSP+) as
giving “developing countries a special incentive to pursue sustainable development and good governance”, in
return for which “the EU cuts its import duties to zero on more than two thirds of the tariff lines of their
exports”.)

7) Consider the following statements regarding Global Partnership on Artificial Intelligence (GPAI).
1. India is a founding member of GPAI.
2. The initiative aims to bridge the gap between theory and practice on AI by supporting cutting-edge
research and applied activities on AI-related priorities.
3. GPAI summits are held in India every year.
How many of the above statements is/are correct?
a) Only one
b) Only two
c) All three
d) None

Solution: b)

Statement 3 is incorrect.

www.insightsonindia.com 48
INSTA CURRENT AFFAIRS QUIZ
India is a founding member of GPAI, having joined the multi-stakeholder initiative in June 2020. The initiative
aims “to bridge the gap between theory and practice on AI by supporting cutting-edge research and applied
activities on AI-related priorities.”

It also brings together people involved in the fields of science, industry, and civil society, along with governments,
international organisations and academia for greater international cooperation. The first three GPAI summits
were held in Montreal, Paris and Tokyo, respectively.

Source

8) Consider the following statements regarding International Energy Agency (IEA).


1. The International Energy Agency (IEA) is a Paris-based autonomous intergovernmental organisation
that provides policy recommendations, analysis and data on the entire global energy sector.
2. India is a member of IEA.
3. IEA recommends that countries should hold an emergency oil stockpile sufficient to provide 90 days of
import protection.
How many of the above statements are correct?
a) Only one
b) Only two
c) All three
d) None

Solution: b)

Statement 2 is incorrect.

The International Energy Agency (IEA), a Paris-based autonomous intergovernmental organisation in which
India is an ‘Association’ country, recommends that all countries should hold an emergency oil stockpile
sufficient to provide 90 days of import protection.

9) Consider the following statements regarding Initiative on Critical and Emerging Technology (iCET).
1. The Initiative on Critical and Emerging Technologies is a framework agreed upon by India and South
Korea.
2. It focuses on addressing regulatory barriers and aligning export controls for smoother trade and deeper
cooperation in critical areas.
3. It includes cooperation in areas of artificial intelligence, quantum computing, semiconductors and
wireless telecommunication.
How many of the above statements are correct?
a) Only one
b) Only two
c) All three
d) None

Solution: b)

Statement 1 is incorrect.

India and the United States unveiled a roadmap for enhanced collaboration in high-technology areas, with a
focus on addressing regulatory barriers and aligning export controls for smoother trade and “deeper
cooperation” in critical areas. This was part of the Initiative on Critical and Emerging Technology (iCET)
announced by President Joe Biden and Prime Minister Narendra Modi last year.

The Initiative on Critical and Emerging Technologies is a framework agreed upon by India and the U.S. for
cooperation on critical and emerging technologies in areas including artificial intelligence, quantum computing,
semiconductors and wireless telecommunication.
www.insightsonindia.com 49
INSTA CURRENT AFFAIRS QUIZ

10) Consider the following statements.


1. Article 24 of the Indian constitution prohibits the employment of children below the age of 14 years in
any factory or mine or employed in any hazardous employment.
2. India has ratified both fundamental ILO Conventions on Child Labour.
3. National Child Labour Project (NCLP) Scheme implemented by the Union Ministry of Women and Child
Development, aims at rehabilitation of child labourers through District Project Societies.
How many of the above statements is/are correct?
a) Only one
b) Only two
c) All three
d) None

Solution: b)

Statement 3 is incorrect.

Article 24 of the Indian constitution prohibits the employment of children below the age of 14 years in any
factory or mine or employed in any hazardous employment. India ratified both fundamental ILO Conventions
on Child Labour (138 and 182) in 2017.

National Child Labour Project (NCLP) Scheme: Implemented by the Union Ministry of Labour & Employment.
Rehabilitation of child labourers through District Project Societies under the District Magistrate.

11) Consider the following statements regarding Hydrographic survey.


1. A hydrographic survey is a scientific process that involves mapping of physical features of underwater
terrain.
2. However it cannot study the depth and topography of the seabed and water bodies.
3. India had a bilateral agreement with Maldives that allowed collaboration in the hydrographic survey of
Maldivian territorial waters.
How many of the above statements is/are correct?
a) Only one
b) Only two
c) All three
d) None

Solution: b)

Statement 2 is incorrect.

The Maldives has expressed its intention to terminate a bilateral agreement with India that allowed collaboration
in the hydrographic survey of Maldivian territorial waters.

What is a Hydrographic Survey?


A hydrographic survey is a scientific process that involves mapping and measuring the physical features of
underwater terrain. It includes the study of the depth, topography, and characteristics of the seabed and water
bodies.

12) Consider the following statements.


1. The issuance of Indian passports began during the First World War under the Defence of India Act.
2. The League of Nations’ 1920 conference aimed to standardize passport regulations, influencing
the common British system.
3. Before India’s Independence, the British government never mandated passports for International
travel.
How many of the above statements is/are correct?
www.insightsonindia.com 50
INSTA CURRENT AFFAIRS QUIZ
a) Only one
b) Only two
c) All three
d) None

Solution: b)

Statement 3 is incorrect.

Passports have historical roots, with mentions in the Hebrew Bible and established systems in countries like
France and the UK. Modern passports emerged due to the British Nationality and Status of Aliens Act in 1914,
featuring photographs and distinguishing features. The League of Nations’ 1920 conference aimed to standardize
passport regulations, influencing the common British system. In the 1920s, the US enacted immigration laws tied
to passports, limiting inflows. Despite initial discomfort, passports have endured as a vital aspect of modern
citizenship.

Passports in India:
The issuance of Indian passports began during the First World War (1914-1918) under the Defence of India Act,
when the British government mandated passports for travel.

Source

13) Consider the following statements regarding International Organization for Migration.
1. It is the principal United Nations agency working in the field of migration.
2. The organization implements operational assistance programmes for migrants.
3. It does not deal with internally displaced persons and refugees.
How many of the above statements is/are correct?
a) Only one
b) Only two
c) All three
d) None

Solution: b)

Statement 3 is incorrect.

International Organization for Migration (founded 1951; HQ: Grand-Saconnex, Switzerland) is the principal United
Nations agency working in the field of migration. The organization implements operational assistance
programmes for migrants, including internally displaced persons, refugees, and migrant workers.

14) Consider the following statements regarding Colombo Security Conclave.


1. Colombo Security Conclave is a trilateral maritime security grouping consisting of India, Sri Lanka and
Thailand.
2. It serves as India’s outreach to the Indian Ocean and aims to counter China’s influence in the region.
3. Colombo Security Conclave does not focus on countering terrorism and radicalisation.
How many of the above statements is/are correct?
a) Only one
b) Only two
c) All three
d) None

Solution: a)

Only Statement 2 is correct.

www.insightsonindia.com 51
INSTA CURRENT AFFAIRS QUIZ

15) Consider the following statements regarding OPEC.


1. OPEC is an intergovernmental organization established at the Baghdad Conference.
2. Angola, Ecuador and Qatar are the member countries of OPEC.
3. The organization’s primary objective is to coordinate and unify petroleum policies among member
countries.
4. OPEC possess over 80% of the world’s total crude oil reserves.
How many of the above statements is/are correct?
a) Only one
b) Only two
c) Only three
d) All four

Solution: c)

Statement 2 is incorrect.

Angola leaves OPEC, a setback for the Saudi-led oil producer group.

OPEC was established in 1960 at the Baghdad Conference by Iran, Iraq, Kuwait, Saudi Arabia, and Venezuela, is
an intergovernmental organization with key members including Algeria, Equatorial Guinea, Gabon, Libya, Nigeria,
the Republic of the Congo, and the United Arab Emirates.

The organization’s primary objective is to coordinate and unify petroleum policies among member countries,
ensuring fair and stable prices for petroleum producers and maintaining an efficient, economic, and regular
www.insightsonindia.com 52
INSTA CURRENT AFFAIRS QUIZ
supply of petroleum to consuming nations. With its headquarters in Vienna, Austria, OPEC holds significant
global influence, possessing over 80% of the world’s total crude oil reserves.

16) Consider the following acts.


1. Killing members of the group
2. Causing serious bodily or mental harm to members of the group
3. Imposing measures intended to prevent births within the group
4. Forcibly transferring children of the group to another group
According to the Convention on the Prevention and Punishment of the Crime of Genocide, how many of the
above acts on a national, ethnical or religious group is/are not considered as Genocide?
a) Only two
b) Only three
c) All four
d) None

Solution: d)

In the present Convention, genocide means any of the following acts committed with intent to destroy, in whole
or in part, a national, ethnical, racial or religious group, as such:
(a) Killing members of the group;
(b) Causing serious bodily or mental harm to members of the group;
(c) Deliberately inflicting on the group conditions of life calculated to bring about its physical destruction in whole
or in part;
(d) Imposing measures intended to prevent births within the group;
(e) Forcibly transferring children of the group to another group.

Source

7. Polity
1) Consider the following statements.
1. Article 22 of the Constitution upholds the fundamental right of any person who is in custody to be
informed as to why he has been arrested.
2. As per the present rules, the Enforcement Directorate (ED) need to mandatorily furnish a person
accused under the Prevention of Money Laundering Act (PMLA) with a copy of the grounds of detention at the
time of his arrest.
Which of the above statements is/are correct?
a) 1 only
b) 2 only
c) Both 1 and 2
d) Neither 1 nor 2

Solution: a)

The Supreme Court has clarified in a judgment that the Enforcement Directorate (ED) need not furnish a
person accused under the Prevention of Money Laundering Act (PMLA) with a copy of the grounds of
detention at the time of his arrest.

The judgment comes amid debate that non-communication of the grounds of arrest was a violation of Article
22(1) of the Constitution, which upholds the fundamental right of any person who is in custody to be informed
as to why he has been arrested.

www.insightsonindia.com 53
INSTA CURRENT AFFAIRS QUIZ
Source

2) Consider the following statements.


1. Federalism allows for the distribution of powers between the federal and state levels.
2. Asymmetric Federalism is based on unequal powers and relationship in political, administrative and
fiscal arrangements.
3. We do not see any form of Asymmetric Federalism in India.
How many of the above statements is/are correct?
a) Only one
b) Only two
c) All three
d) None

Solution: b)

Statement 3 is incorrect.

www.insightsonindia.com 54
INSTA CURRENT AFFAIRS QUIZ
3) Consider the following statements regarding Subordinate Legislations.
1. Subordinate legislations also known as delegated legislations are laws created by authorities
to implement and administer the broader statutes passed by the legislature.
2. The concept of delegated legislation is mentioned specifically in the Indian Constitution.
3. Rajya Sabha does not have a Subordinate Legislation Committee.
How many of the above statements is/are correct?
a) Only one
b) Only two
c) All three
d) None

Solution: a)

Only Statement 1 is correct.

The Rajya Sabha’s Subordinate Legislation Committee has criticized the Union government for prolonged delays
in framing rules for Acts passed by Parliament.

What are Subordinate Legislations?


Subordinate legislations are laws or regulations created by authorities to implement and administer the broader
statutes passed by the legislature. These regulations, also known as delegated or secondary legislation, provide
detailed guidelines and procedures necessary for the effective enforcement of primary laws.
Although the concept of delegated legislation was not mentioned specifically in the Indian Constitution it can be
understood by interpreting Article 312.

Source

4) Consider the following statements regarding Committee on Subordinate Legislation in Rajya Sabha.
1. It is formed as per the Rules of Procedure and Conduct of Business in the Council of States.
2. Its mandate involves scrutinizing and reporting to the House on the proper exercise of powers to make
rules, regulations, and bylaws as conferred by the Constitution.
3. The parliament cannot delegate to the committee to frame rules.
How many of the above statements is/are correct?
a) Only one
b) Only two
c) All three
d) None

Solution: b)

Statement 3 is incorrect.

About the Committee on Subordinate Legislation in Rajya Sabha


It is formed under Rule 204 of the Rules of Procedure and Conduct of Business in the Council of States. Its
mandate involves scrutinizing and reporting to the House on the proper exercise of powers to make rules,
regulations, bylaws, schemes, etc., as conferred by the Constitution or delegated by Parliament.

5) Right to free legal aid, Right to fair trial, Right to hearing and Right to speedy trial and are provided to citizens
under which of the following fundamental rights?
a) Right against arbitrary arrest and detention
b) Right against exploitation
c) Protection of Life and Personal Liberty
d) Equality before law and equal protection of laws

Solution: c)
www.insightsonindia.com 55
INSTA CURRENT AFFAIRS QUIZ

The Supreme Court has reaffirmed its judgement in the Menaka case in the subsequent cases. It has declared the
following rights as part of Article 21:
(1) Right to live with human dignity.
(2) Right to decent environment including pollution free water and air and protection against hazardous
industries.
(3) Right to livelihood.
(4) Right to privacy.
(5) Right to shelter.
(6) Right to health.
(7) Right to free education up to 14 years of age.
(8) Right to free legal aid.
(9) Right against solitary confinement.
(10) Right to speedy trial.
(11) Right against handcuffing.
(12) Right against inhuman treatment.
(13) Right against delayed execution.
(14) Right to travel abroad.
(15) Right against bonded labour.
(16) Right against custodial harassment.
(17) Right to emergency medical aid.
(18) Right to timely medical treatment in government hospital.
(19) Right not to be driven out of a state.
(20) Right to fair trial.
(21) Right of prisoner to have necessities of life.
(22) Right of women to be treated with decency and dignity.
(23) Right against public hanging.
(24) Right to hearing.
(25) Right to information.
(26) Right to reputation.
(27) Right of appeal from a judgement of conviction
(28) Right to social security and protection of the family
(29) Right to social and economic justice and empowerment
(30) Right against bar fetters
(31) Right to appropriate life insurance policy
(32) Right to sleep
(33) Right to freedom from noise pollution
(34) Right to electricity

6) Consider the following statements.


1. All India Services are governed by Article 312 of the Constitution of India.
2. Indian Administrative Service, the Indian Police Service and the Indian Foreign Service are the three All
India Services.
3. Central Civil Services are under the central government itself with no state cadre system.
How many of the above statements are correct?
a) Only one
b) Only two
c) All three
d) None

Solution: b)

Statement 2 is incorrect.

www.insightsonindia.com 56
INSTA CURRENT AFFAIRS QUIZ
There are three All India Services – the Indian Administrative Service, the Indian Police Service and the Indian
Forest Service – which are selected by the central government with officers allotted to various state cadres. The
Centre then gets a certain percentage of officers from each state on central deputation. These bureaucrats work
directly for the Centre. All India Services are governed by Article 312 of the Constitution of India.

Other services are called Central Civil Services. These services are under the central government itself with no
state cadre system. They include services such as the Indian Foreign Service, the Indian Revenue Service, Customs
and Central Excise Service and several others.

7) Consider the following statements.


1. The size of the State Legislative Council cannot exceed one third of the membership of the State
Legislative Assembly, with a lower cap of 40 members.
2. The total number of Ministers, including the Prime Minister, in the Council of Ministers shall not exceed
fifteen per cent of the total number of members of the House of the People.
Which of the above statements is/are incorrect?
a) 1 only
b) 2 only
c) Both 1 and 2
d) Neither 1 nor 2

Solution: d)

• The size of the State Legislative Council cannot be more than one third of the membership of the State
Legislative Assembly. However, its size cannot be less than 40 members.
• Before the 91st Amendment Act (2003), the convention of appointing a lot of ministers on the council due to
political reasons led to widespread abuse of the system.
• It led to very large size of the Council of Ministers. Besides, when no party had a clear majority, there was a
temptation to win over the support of the members of the Parliament by giving them ministerial positions as
there was no restriction on the number of the members of the Council of Ministers.
• This was happening in many States also. Therefore, an amendment was made that the Council of Ministers
shall not exceed 15 percent of total number of members of the House of People (or Assembly, in the case of
the States).

8) Article 2 of the constitution empowers the Parliament to


1. Admit into the Union of India new states
2. Increase of the area of any state
3. Alter the boundaries of any state
How many of the above statements are correct?
a) Only one
b) Only two
c) All three
d) None

Solution: a)

Article 2 of the Constitution grants two powers to the parliament: the power to admit into the Union new states
and the power to establish new states. It does not deal with the formation of or changes in the existing states of
the Union. Article 3 deals with the internal re-adjustments of the territories. Hence, the increase in area of any
state and altering the boundaries of states comes under the purview of Article 3.

9) Consider the following statements.


1. The Constitution of India stipulates that a state cannot raise a loan without the consent of the
Government of India if any part of a previous loan from the central government remains outstanding.
2. States in India have a normal net borrowing ceiling set at 5% of Gross State Domestic Product
(GSDP) for the fiscal year 2024, as per the Fifteenth Finance Commission’s recommendation.
www.insightsonindia.com 57
INSTA CURRENT AFFAIRS QUIZ
3. States in India receive an extra 0.5% of GSDP in borrowing capacity as a performance-based
incentive for power sector reforms.
How many of the above statements is/are correct?
a) Only one
b) Only two
c) All three
d) None

Solution: b)

Statement 2 is incorrect.

Rules regarding borrowing by States:


• States in India have a normal net borrowing ceiling set at 3% of Gross State Domestic Product (GSDP) for the
fiscal year 2024, as per the Fifteenth Finance Commission’s recommendation. Additionally, states receive an
extra 0.5% of GSDP in borrowing capacity as a performance-based incentive for power sector reforms,
enabling them to borrow ₹1.43 lakh crore for FY24 based on the Ministry of Power’s recommendation.
• Constitutionally, Article 293(3) stipulates that a state cannot raise a loan without the consent of the
Government of India if any part of a previous loan from the central government remains outstanding.

Source

10) Consider the following statements regarding Parliamentary Committees in India.


1. Joint Parliamentary Committees (JPCs) and Select Committees are usually chaired by ruling party MPs,
and are disbanded after they have submitted their report.
2. There are equal number of Departmentally Related Standing Committees for Lok Sabha and Rajya
Sabha and are headed by members of these respective Houses.
3. Committee meetings are independent of Parliament’s calendar.
How many of the above statements is/are correct?
a) Only one
b) Only two
c) All three
d) None

Solution: b)

Statement 2 is incorrect.

Parliament can also constitute a Joint Parliamentary Committee (JPC) with a special purpose, with members from
both Houses, for detailed scrutiny of a subject or Bill. Also, either of the two Houses can set up a Select
Committee with members from that House. JPCs and Select Committees are usually chaired by ruling party MPs,
and are disbanded after they have submitted their report.

The time to speak on a Bill is allocated according to the size of the party in the House. MPs often do not get
adequate time to put forward their views in Parliament, even if they are experts on the subject. Committees are
small groups with relatively less demands on their time; in these meetings, every MP gets a chance and the time
to contribute to the discussion. Parliament has only around 100 sittings a year; Committee meetings are
independent of Parliament’s calendar.

There are 16 Departmentally Related Standing Committees for Lok Sabha and eight for Rajya Sabha; however,
every Committee has members from both Houses. Lok Sabha and Rajya Sabha panels are headed by members of
these respective Houses.

11) Consider the following statements.


1. The Chamber of Princes is the Library Hall in the Old Parliament.
www.insightsonindia.com 58
INSTA CURRENT AFFAIRS QUIZ
2. The Chamber of Princes was established by a Royal Proclamation of King Emperor George V to
advocate the common interests of British India and the princely states.
3. The Federal Court of India began functioning from the Chamber of Princes, and later the Supreme
Court of India was inaugurated here.
How many of the above statements are correct?
a) Only one
b) Only two
c) All three
d) None

Solution: c)

In the 1919 plan for the construction of the Parliament, it was decided to have a council house, comprising a
Legislative Assembly Chamber (which later became the Lok Sabha), a Council of States Chamber (which is now
the Rajya Sabha) and the Chamber of Princes.

The Chamber of Princes is the Library Hall in the Old Parliament.


The Chamber of Princes, also known as Narendra Mandal, was established in 1920 by a Royal Proclamation of
King Emperor George V to advocate the common interests of British India and the princely states.

In 1937, the Federal Court of India began functioning from the Chamber of Princes.
It was 12 years later that in this building, the Supreme Court of India was inaugurated. “The inaugural
proceedings were simple but impressive,” according to the official account of the Supreme Court.

12) Article 299 of the Constitution provides that


1. All contracts made in the exercise of the executive power of the Union or of a State shall be expressed
to be made by the President or by the Governor of the State.
2. Neither the President nor the Governor can be personally held liable for such contracts.
3. Contracts not adhering to the manner given in the Article cannot be enforced by any contracting party.
How many of the above statements are correct?
a) Only one
b) Only two
c) All three
d) None

Solution: c)

Article 298 grants the Centre and the state governments the power to carry on trade or business, acquire, hold,
and dispose of property, and make contracts for any purpose, while Article 299 delineates the manner in which
these contracts will be concluded. Articles 298 and 299 came after the Constitution came into effect and the
government entered into contracts even in the pre-independence era. According to the Crown Proceedings Act of
1947, the Crown could not be sued in court for a contract it entered into.

Article 299 of the Constitution provides that “all contracts made in the exercise of the executive power of the
Union or of a State shall be expressed to be made by the President or by the Governor of the State” and that all
such contracts and “assurances of property made in the exercise of that power shall be executed” on behalf of
the President or the Governor by persons in a manner as directed and authorised by them.

The objective behind Article 299(1), as per the 1954 top court ruling in ‘Chatturbhuj Vithaldas Jasani v.
Moreshwar Parashram & Ors’, is that there must be a definite procedure according to which contracts must be
made by agents acting on the government’s behalf; otherwise, public funds may be depleted by unauthorized or
illegitimate contracts. It implies that contracts not adhering to the manner given in Article 299(1) cannot be
enforced by any contracting party.

www.insightsonindia.com 59
INSTA CURRENT AFFAIRS QUIZ
However, Article 299 (2) says that essentially, neither the President nor the Governor can be personally held
liable for such contracts.

8. History, Art and Culture


1) "Nilakkanavu", envisioned as a tribute to the scientists who contributed to the success of Chandrayaan
Mission was recently adapted into which of the following classical dance form?
a) Bharatanatyam
b) Kathak
c) Kathakali
d) Mohiniyattam

Solution: d)

2) Consider the following statements.


1. Jallikattu, is a bull-taming sport traditionally played in Tamil Nadu as part of the Navaratri festival.
2. The Animal Welfare Board of India is a statutory advisory body established under Wild Life (Protection)
Act, 1972.
3. Prevention of cruelty to animals is included in the List III of the Seventh Schedule to the Constitution of
India.
How many of the above statements is/are correct?
a) Only one
b) Only two
c) All three
d) None

Solution: a)
www.insightsonindia.com 60
INSTA CURRENT AFFAIRS QUIZ

Only Statement 3 is correct.

Jallikattu, also known as eruthazhuvuthal, is a bull-taming sport traditionally played in Tamil Nadu as part of the
Pongal harvest festival. The festival is a celebration of nature, and thanksgiving for a bountiful harvest, of which
cattle-worship is part.

The Animal Welfare Board of India is a statutory advisory body on Animal Welfare Laws and promotes animal
welfare in the country. Established in 1962 under Section 4 of the Prevention of Cruelty to Animals Act, 1960.

Entry 17 of List III of the Seventh Schedule to the Constitution which relates to the prevention of cruelty to
animals.

3) Consider the following statements regarding Uttaramerur inscription.


1. Uttaramerur inscription is found in Kanchipuram, Tamil Nadu.
2. These provide a detailed description about the village’s self-governance.
3. Uttaramerur is known for its historic temples built during Pallava and Chola rule.
How many of the above statements is/are correct?
a) Only one
b) Only two
c) All three
d) None

Solution: c)

While Uttaramerur has multiple inscriptions spanning centuries, the most famous one is from the reign of
Parantaka I (907-953 AD). These provide a detailed description about the village’s self-governance and have
been cited by historians and political leaders alike as evidence of India’s history of democratic functioning.

Uttaramerur lies in present-day Kanchipuram district, approximately 90 km southeast of Chennai. It is known for
its historic temples built during Pallava and Chola rule.

The inscription found there is like a local constitution for the gram sabha. It tells how the assembly should be run,
what should be the qualification of members, what should be the process to elect the members, and how a
member would be disqualified.

4) Consider the following statements regarding prehistoric Dancing Girl sculpture.


1. The Dancing Girl figurine was discovered in Mohenjodaro after India’s independence.
2. The sculpture was made in lost-wax casting method.
3. Even though Mohenjodaro and Harappa became part of Pakistani territory after the Partition, the
Dancing Girl remained in India as part of an agreement.
How many of the above statements are correct?
a) Only one
b) Only two
c) All three
d) None

Solution: b)

Statement 1 is incorrect.

The Dancing Girl figurine was discovered in Mohenjodaro in 1926.

Even though Mohenjodaro and Harappa became part of Pakistani territory after the Partition, the Dancing Girl
remained in India as part of an agreement. Today, the bronze figurine sits in the National Museum of India.
www.insightsonindia.com 61
INSTA CURRENT AFFAIRS QUIZ

The Dancing Girl is evidence of the civilisation’s knowledge of metal blending and lost-wax casting – a
complicated process by which a duplicate sculpture is cast from an original sculpture to create highly detailed
metallic artefacts.

5) Consider the following statements regarding Konsachem.


1. It is an important harvest festival celebrated mainly in Kerala.
2. Both Hindus and Catholics celebrate in their unique ways.
3. The festival involves blessing the first sheaves of rice when harvested.
How many of the above statements is/are correct?
a) Only one
b) Only two
c) All three
d) None

Solution: b)

Statement 1 is incorrect.

What is Konsachem?
It is an important harvest festival celebrated in Goa, primarily in August. Both Hindus and Catholics participate
in their unique ways. The festival involves blessing the first sheaves of rice when harvested.

Source

9. Agriculture
1) Consider the following statements regarding Cropping pattern.
1. It can be defined as the proportion of area under various crops at a point of time.
2. It is a dynamic concept because it changes over space and time.
Which of the above statements is/are incorrect?
a) 1 only
b) 2 only
c) Both 1 and 2
d) Neither 1 nor 2

Solution: d)

Cropping pattern: It is a dynamic concept because it changes over space and time. It can be defined as the
proportion of area under various crops at a point of time. In other words, it is a yearly sequence and spatial
arrangement of sowing and fallow on a given area. In India, the cropping pattern determined by rainfall, climate,
temperature, soil type and technology.

2) Which of the following is/are the advantages of Indian Raw Sugar in the International Market?
1. Season advantage
2. Indian sugar has a high amount of dextran
3. High Sucrose content
How many of the above statements is/are correct?
a) Only one
b) Only two
c) All three
d) None

www.insightsonindia.com 62
INSTA CURRENT AFFAIRS QUIZ

Solution: b)

Statement 2 is incorrect.

Advantages of Indian raw sugar in the International Market:


• Season advantage: India’s crushing is from October to April, whereas Brazilian mills operate from April to
November. Hence, importers are utilising Indian raw sugar during Brazil’s off-season.
• Freight cost savings: India is much closer to its major sugar importers (compared to Brazil) and therefore less
freight cost.
• Indian sugar is free of dextran: Dextran is a bacterial compound formed when sugarcane stays in the sun for
too long after harvesting.
• Indian raw sugar has no dextran, as it is produced from fresh cane crushed within 12-24 hours of harvesting
(compared to 48 hours or more in Brazil) India’s sugar has a higher % of sucrose present.

3) Consider the following statements regarding Wheat.


1. In terms of production, Wheat is the most significant cereal crop in India.
2. Wheat is a kharif crop.
3. Ganga-Satluj plains and black soil region of Deccan is suitable for Wheat cultivation.
How many of the above statements is/are correct?
a) Only one
b) Only two
c) All three
d) None

Solution: a)

Only Statement 3 is correct.

4) Consider the following statements regarding Mixed Cropping:


1. When two or more crops are grown on an equivalent land simultaneously, it’s referred to as mixed
cropping.
2. The practice of this method helps to minimize the risk of the failure of one of the crops.
3. The crops that are grown together should have a different maturation time and different water
requirements.
How many of the above statements are correct?
a) Only one
b) Only two
c) All three
d) None

www.insightsonindia.com 63
INSTA CURRENT AFFAIRS QUIZ
Solution: c)

Mixed Cropping: When two or more crops are grown on an equivalent land simultaneously, it’s referred to as
mixed cropping. For example, growing wheat and gram on an equivalent land at an equivalent time is mixed
cropping.
The practice of this method helps to minimize the risk of the failure of one of the crops and provides insurance
against the crop failure due to abnormal weather conditions. The crops that are grown together should have a
different maturation time and different water requirements.

5) Consider the following statements regarding Agriculture Census


1. It is a decadal census.
2. The basic unit of data collection in Agriculture Census is the operational holding.
3. In India, the Department of Agriculture and Farmers Welfare has been organizing Agricultural Census in
collaboration with the States and Union Territories.
How many of the above statements is/are correct?
a) Only one
b) Only two
c) All three
d) None

Solution: b)

Statement 1 is incorrect.

Agriculture Census in India is conducted every five years. In India, the Department of Agriculture and Farmers
Welfare has been organizing Agricultural Census, since 1970-71 in collaboration with the States and Union
Territories as part of programme of World Census of Agriculture.

Through Agriculture Census, basic data on important aspects of agricultural economy of operational holdings in
the country is collected.
The basic unit of data collection in Agriculture Census is the operational holding.

6) Consider the following statements regarding Electronic soil’ (eSoil).


1. eSoil is an electrically conductive growth substrate derived from cellulose and a conductive polymer
(PEDOT).
2. eSoil works by electrically stimulating plant roots.
3. It is not compatible with hydroponics.
How many of the above statements is/are correct?
a) Only one
b) Only two
c) All three
d) None

Solution: b)

Statement 3 is incorrect.

• Researchers introduced a ground-breaking study on “electronic soil” (eSoil) in hydroponics, offering the
potential for sustainable urban agriculture and food security.
• eSoil is an electrically conductive growth substrate derived from cellulose and a conductive polymer
(PEDOT).
• It stimulates plant roots and the growth environment with low power, providing a safer and environmentally
friendly alternative to traditional methods.
• eSoil works by electrically stimulating plant roots.

www.insightsonindia.com 64
INSTA CURRENT AFFAIRS QUIZ
• This promotes effective and sustainable development, expanding the range of crops that can be grown
hydroponically—without soil, using only water, nutrients, and a substrate.
• The benefits of eSoil include low energy consumption, safety, and its compatibility with hydroponics,
enabling urban agriculture to meet the challenges of a growing population and climate change.

Source

10. Defence and Security


1) Which of the following are the submarines of Indian Navy?
1. INS Kalvari
2. INS Vikrant
3. INS Arihant
4. INS Vikramaditya
How many of the above options is/are correct?
a) Only one
b) Only two
c) Only three
d) All four

Solution: b)

Option 1 and 3 is correct.

www.insightsonindia.com 65
INSTA CURRENT AFFAIRS QUIZ

11. Reports and Indices


1) ‘The Forest Pathways Report’ was published recently by:
a) UNEP
b) IUCN
c) World Wildlife Fund
d) UN Forum on Forests

Solution: c)

A startling increase in the speed and intensity of global deforestation has derailed efforts to protect and restore
forests by 2030, according to two new reports analyzing progress toward global forest conservation goals.

WWF's Forest Pathways 2023 report and the Forest Declaration Assessment detail the immense scale of forest
loss just two years after more than 130 countries representing 85% of the planet's forests pledged to halt and
reverse deforestation by the end of the decade. The lack of progress on commitments leaves the world in clear
danger of missing vital targets.

2) International Debt Report (IDR), that was recently in news was released by

www.insightsonindia.com 66
INSTA CURRENT AFFAIRS QUIZ
a) International Monetary Fund
b) Asian Development Bank (ADB)
c) World Bank
d) World Trade Organisation

Solution: c)

The World Bank released the International Debt Report (IDR).

What are debts?


Debt refers to an obligation or financial liability that one party owes to another. It is typically in the form of
borrowed funds that need to be repaid over time, often with interest.

India’s total debt is around 81%, but most of it is domestic debt. India’s external debt is around 18.6% (around
USD 624 billion at the end of March 2023), as per the RBI data.

Source

12. Maps / Places


1) The Strait of Bab el-Mandeb, connects
a) Persian Gulf and Gulf of Aden
b) Mediterranean Sea and Red Sea
c) Red Sea and Gulf of Aden
d) Suez Canal and Red Sea

Solution: c)

The Bab-el-Mandeb is a strait between Yemen on the Arabian Peninsula and Djibouti and Eritrea in the Horn of
Africa. It connects the Red Sea to the Gulf of Aden.

Source

2) Consider the following statements.


1. Ennore Creek is situated along the Malabar Coast.
2. It acts as natural shock absorbers during calamities.
3. The creek is part of the Pulicat water system.
How many of the above statements is/are correct?

www.insightsonindia.com 67
INSTA CURRENT AFFAIRS QUIZ
a) Only one
b) Only two
c) All three
d) None

Solution: b)

Statement 1 is incorrect.

Ennore Creek in Chennai, situated along the Coromandel Coast, serves as a crucial buffer for the aquifers of
the Araniyar-Kosasthalaiyar Basin, protecting them from the sea. Positioned in the floodplains of three rivers on
a disaster-prone coastline, Chennai relies on wetlands like Ennore Creek as natural shock absorbers during
calamities.

The creek is part of the Pulicat water system, including the Pulicat lagoon and the Buckingham Canal.

Source

3) Mount Etna Is an active volcano located in


a) Congo
b) Philippines
c) Hawaii
d) Italy

Solution: d)

Europe’s most active volcano Mount Etna erupted on May 21, 2023.

Mount Etna is in an almost constant state of activity and it’s considered a United Nations Educational, Scientific
and Cultural Organization World Heritage Site.
The volcano’s height increased by over 30 metres following a six-month period of activity. This sudden growth
spurt is the result of about 50 eruptions at the southeastern crater since February 16, 2021. The height grew due
to the amount of volcanic material it spewed.

4) Consider the following statements.


1. Yemen is located at the northern end of the Arabian Peninsula.
2. Yemen does not share its border with Saudi Arabia and Oman.
3. Yemen is a member of United Nations and Non-Aligned Movement.
www.insightsonindia.com 68
INSTA CURRENT AFFAIRS QUIZ
How many of the above statements is/are correct?
a) Only one
b) Only two
c) All three
d) None

Solution: a)

Only Statement 3 is correct.

Yemen is a country in West Asia located at the southern end of the Arabian Peninsula. It shares borders
with Saudi Arabia and Oman and maritime borders with Eritrea, Djibouti, and Somalia. Yemen is a member of
the Arab League, the United Nations, the Non-Aligned Movement, and the Organisation of Islamic Cooperation.

5) Mount Marapi, that was recently in news is located in


a) Kenya
b) Italy
c) Indonesia
d) Peru

Solution: c)

Indonesia's Mount Marapi erupted again recently.

Marapi is known for sudden eruptions that are difficult to predict because they are not caused by a deep
movement of magma, which sets off tremors that register on seismic monitors.

Source

6) Consider the following statements.


1. Bhutan is a landlocked country in the Eastern Himalayas.
2. Bhutan has a constitutional monarchy with a king as the head of state and without any prime minister.
3. Vajrayana Buddhism is the state religion of Bhutan.
www.insightsonindia.com 69
INSTA CURRENT AFFAIRS QUIZ
How many of the above statements is/are correct?
a) Only one
b) Only two
c) All three
d) None

Solution: b)

Statement 2 is incorrect.

Bhutan is a landlocked country in the Eastern Himalayas, bordered by China and India. Known as “Druk Yul” or
“Land of the Thunder Dragon,” it has a constitutional monarchy with a king as the head of state and a prime
minister as the head of government. Vajrayana Buddhism is the state religion, and Gangkhar Puensum is its
highest peak, also the highest unclimbed mountain globally.

7) Arrange the following from North to South.


1. Veimandu Channel
2. Kardiya Channel
3. Equatorial Channel
Select the correct answer code:
a) 2-1-3
b) 1-2-3
c) 2-3-1
d) 3-2-1

Solution: a)

www.insightsonindia.com 70
INSTA CURRENT AFFAIRS QUIZ

www.insightsonindia.com 71

You might also like